OB EXAM 1

Pataasin ang iyong marka sa homework at exams ngayon gamit ang Quizwiz!

In providing counseling to pregnant women, the nurse can include information on the components of weight gain during pregnancy and the amount of weight that will be lost after the birth. Match the tissues contributing to maternal weight gain at 40 weeks of gestation with the approximate amount of weight gain in kilograms. a. 0.5 to 1.8 b. 1.8 to 2.3 c. 3.2 to 3.9 d. 0.9 e. 0.9 to 1.1 1. Fetus 2. Placenta 3. Breast tissue 4. Amniotic fluid 5. Blood volume

1. C 2. E 3. A 4. D 5. B

All pregnant women should be instructed to recognize and report potential complications for each trimester of pregnancy. Match the sign or symptom with a possible cause. a. Severe vomiting in early pregnancy b. Epigastric pain in late pregnancy c. Severe backache and flank pain d. Decreased fetal movement e. Glycosuria 1. Fetal jeopardy or intrauterine fetal death 2. Kidney infection or stones 3. Gestational diabetes 4. Hyperemesis gravidarum 5. Hypertension, preeclampsia

1. D 2. C 3. E 4. A 5. B

To promote wellness and prevent illness throughout the life span, it is important for the nurse to be cognizant of immunization recommendations for women older than 18 years. Match each immunization with the correct schedule. a. Tetanus-diphtheria-pertussis (Tdap) b. Measles, mumps, rubella c. Herpes zoster d. Hepatitis B e. Influenza f. Human papillomavirus (HPV) 1. Three injections for girls between the ages 9 to 26 years 2. Primary series of three injections Annually 3. Once and then a booster every 10 year 4. One dose after age 65 years 5. Once if born after 1956

1. f 2. d 3. e 4. a 5. c 6. b

Due to the effects of cyclic ovarian changes in the breast, when is the best time for breast self-examination (BSE)? a. Between 5 and 7 days after menses ceases b. Day 1 of the endometrial cycle c. Midmenstrual cycle d. Any time during a shower or bath

A The physiologic alterations in breast size and activity reach their minimal level approximately 5 to 7 days after menstruation ceases. Therefore, BSE is best performed during this phase of the menstrual cycle. Day 1 of the endometrial cycle is too early to perform an accurate BSE. After the midmenstrual cycle, breasts are likely to become tender and increase in size, which is not the ideal time to perform BSE. Lying down after a shower or bath with a small towel under the shoulder of the side being examined is appropriate teaching for BSE. A secondary BSE may be performed while in the shower.

What is important for the nurse to recognize regarding the new father and his acceptance of the pregnancy and preparation for childbirth? a. The father goes through three phases of acceptance of his own. b. The father's attachment to the fetus cannot be as strong as that of the mother because it does not start until after the birth. c. In the last 2 months of pregnancy, most expectant fathers suddenly get very protective of their established lifestyle and resist making changes to the home. d. Typically, men remain ambivalent about fatherhood right up to the birth of their child.

ANS: A A father typically goes through three phases of development to reach acceptance of fatherhood: the announcement phase, the moratorium phase, and the focusing phase. The father-child attachment can be as strong as the mother-child relationship and can also begin during pregnancy. During the last 2 months of the pregnancy, many expectant fathers work hard to improve the environment of the home for the child. Typically, the expectant father's ambivalence ends by the first trimester, and he progresses to adjusting to the reality of the situation and then to focusing on his role.

The nurse has formulated a diagnosis of Inadequate Nutrition for the client. Which goal is most appropriate for this client to obtain? a. Gain a total of 30 pounds. b. Consistently take daily supplements. c. Decrease her intake of snack foods. d. Increase her intake of complex carbohydrates.

ANS: A A weight gain of 30 pounds is one indication that the client has gained a sufficient amount for the nutritional needs of pregnancy. A daily supplement is not the best goal for this client and does not meet the basic need of proper nutrition during pregnancy. Decreasing snack foods may be needed and should be assessed; however, assessing weight gain is the best method of monitoring nutritional intake for this pregnant client. Although increasing the intake of complex carbohydrates is important for this client, monitoring the weight gain should be the end goal.

Which information regarding protein in the diet of a pregnant woman is most helpful to the client? a. Many protein-rich foods are also good sources of calcium, iron, and B vitamins. b. Many women need to increase their protein intake during pregnancy. c. As with carbohydrates and fat, no specific recommendations exist for the amount of protein in the diet. d. High-protein supplements can be used without risk by women on macrobiotic diets.

ANS: A Good sources for protein, such as meat, milk, eggs, and cheese, have a lot of calcium and iron. Most women already eat a high-protein diet and do not need to increase their intake. Protein is sufficiently important that specific servings of meat and dairy are recommended. High-protein supplements are not recommended because they have been associated with an increased incidence of preterm births.

A pregnant woman reports that she is still playing tennis at 32 weeks of gestation. Which recommendation would the nurse make for this client after a tennis match? a. Drink several glasses of fluid. b. Eat extra protein sources such as peanut butter. c. Enjoy salty foods to replace lost sodium. d. Consume easily digested sources of carbohydrate.

ANS: A If no medical or obstetric problems contraindicate physical activity, then pregnant women should get 30 minutes of moderate physical exercise daily. Liberal amounts of fluid should be consumed before, during, and after exercise because dehydration can trigger premature labor. The woman's caloric intake should be enough to meet the increased needs of pregnancy and the demands of exercise.

A new mother asks the nurse about the "white substance" covering her infant. How should the nurse explain the purpose of vernix caseosa? a. Vernix caseosa protects the fetal skin from the amniotic fluid. b. Vernix caseosa promotes the normal development of the peripheral nervous system. c. Vernix caseosa allows the transport of oxygen and nutrients across the amnion. d. Vernix caseosa regulates fetal temperature.

ANS: A Prolonged exposure to the amniotic fluid during the fetal period could result in the breakdown of the skin without the protection of the vernix caseosa. Normal development of the peripheral nervous system was dependent on nutritional intake of the mother. The amnion was the inner membrane that surrounded the fetus and was not involved in the oxygen and nutrient exchange. The amniotic fluid helped maintain fetal temperature.

Which comment by the nurse would be most helpful in instruct the woman on how best to reduce her risk of developing toxic shock syndrome (TSS)? (Select all that apply.) a. "You should always leave your diaphragm in place for at 6 hours after intercourse." b. "You should remove your diaphragm right after intercourse to prevent TSS." c. "You can use your diaphragm during your menstrual cycle as long as you clean it well." d. "Make sure you don't leave your diaphragm in for longer than 24 hours" e. Report a fever of 38.4 0 C to your health care provider immediately."

ANS: A The nurse should instruct the client on the proper use and removal of the diaphragm and include the danger signs of TSS. The diaphragm must remain against the cervix for 6 to 8 hours to prevent pregnancy, but it should not remain in place longer than 8 hours to avoid the risk of TSS. The diaphragm should not be used during menses. A fever of 38.4 C or higher could be a symptom of TSS and should be reported immediately.

What is the primary role of the doula during labor? a. Helps the woman perform breathing techniques and provides support to the woman and her partner b. Checks the fetal monitor tracing for effects of the labor process on the fetal heart rate c. Takes the place of the father as a coach and support provider d. Administers pain medications as needed by the woman

ANS: A A doula is professionally trained to provide labor support, including physical, emotional, and informational support, to both the woman and her partner during labor and the birth. The doula does not become involved with clinical tasks. While the doula provides support, the goal is not to take the place of anyone in the mother's support group.

A couple have arrived for their preprocedural In vitro fertilization-embryo transfer (IVF-ET) interview. Which explanation regarding the procedure is most accurate? a. "The procedure begins with collecting eggs from the woman's ovaries." b. "A donor embryo will be transferred into the woman's uterus." c. "Donor sperm will be used to inseminate the woman." d. "Don't worry about the technical stuff; that's what we are here for."

ANS: A A woman's eggs are collected from her ovaries, fertilized in the laboratory with the partner's sperm, and transferred to her uterus after normal embryonic development has occurred. Transferring a donor embryo to the woman's uterus describes the procedure for a donor embryo. Inseminating the woman with donor sperm describes therapeutic donor insemination. Telling the client not to worry discredits the client's need for teaching and is not the most appropriate response.

A young woman arrives at the emergency department and states that she thinks she has been raped. She is sobbing and expresses disbelief that this could happen because the perpetrator was a very close friend. Which statement is most appropriate nursing response? a. "Rape is not limited to strangers and frequently occurs by someone who is known to the victim." b. "I would be very upset if my best friend did that to me; that is very unusual." c. "You must feel very betrayed. In what way do you think you might have led him on?" d. "This does not sound like rape. Didn't you just change your mind about having sex after the fact?"

ANS: A Acquaintance rape involves individuals who know one another. Sexual assault occurs when the trust of a relationship is violated. Victims may be less prone to recognize what is happening to them because the dynamics are different from those of stranger rape. It is not at all unusual for the victim to know and trust the perpetrator. Stating that the woman might have led the man to attack her indicates that the sexual assault was somehow the victim's fault. This type of mentality is not constructive. Nurses must first reflect on their own feelings and learn to be unbiased when dealing with victims. A statement of this type can be very psychologically damaging to the victim. Nurses must display compassion by first believing what the victim states. The nurse is not responsible for deciphering the facts involving the victim's claim.

Individual irregularities in Nthe RovarIianG(meBn.strCualM) cycle are most often caused by what? a. Variations in the follicular (preovulatory) phase b. Intact hypothalamic-pituitary feedback mechanism c. Functioning corpus luteum d. Prolonged ischemic phase

ANS: A Almost all variations in the length of the ovarian cycle are the result of variations in the length of the follicular phase. This information discounts the other options as being correct. An intact hypothalamic-pituitary feedback mechanism would be regular, not irregular. The luteal phase begins after ovulation. The corpus luteum is dependent on the ovulatory phase and fertilization. During the ischemic phase, the blood supply to the functional endometrium is blocked, and necrosis develops. The functional layer separates from the basal layer, and menstrual bleeding begins.

Which presumptive sign or symptom of pregnancy would a client experience who is approximately 10 weeks of gestation? a. Amenorrhea b. Positive pregnancy test c. Chadwick sign d. Hegar sign

ANS: A Amenorrhea is a presumptive sign of pregnancy. Presumptive signs of pregnancy are those felt by the woman. A positive pregnancy test and the presence of the Chadwick and Hegar signs are all probable signs of pregnancy.

Which information regarding amniotic fluid is important for the nurse to understand? a. Amniotic fluid serves as a source of oral fluid and a repository for waste from the fetus. b. Volume of the amniotic fluid remains approximately the same throughout the term of a healthy pregnancy. c. The study of fetal cells in amniotic fluid yields little information. d. A volume of more than 2 L of amniotic fluid is associated with fetal renal abnormalities.

ANS: A Amniotic fluid serves as a source of oral fluid, serves as a repository for waste from the fetus, cushions the fetus, and helps maintain a constant body temperature. The volume of amniotic fluid constantly changes. The study of amniotic fluid yields information regarding the sex of the fetus and the number of chromosomes. Too much amniotic fluid (hydramnios) is associated with gastrointestinal and other abnormalities.

Which women should undergo prenatal testing for the human immunodeficiency virus (HIV)? a. All women, regardless of risk factors b. Women who have had more than one sexual partner c. Women who have had a sexually transmitted infection (STI) d. Woman who are monogamous with one partner

ANS: A An HIV test is recommended for all women, regardless of risk factors. The incidence of perinatal transmission from an HIV-positive mother to her fetus ranges from 25% to 35%. Women who test positive for HIV can then be treated.

What kind of fetal anomalies are most often associated with oligohydramnios? a. Renal b. Cardiac c. Gastrointestinal d. Neurologic

ANS: A An amniotic fluid volume of less than 300 ml (oligohydramnios) is often associated with fetal renal anomalies. The amniotic fluid volume has no bearing on the fetal cardiovascular system. Gastrointestinal anomalies are associated with hydramnios or an amniotic fluid volume greater than 2 L. The amniotic fluid volume has no bearing on the fetal neurologic system.

A woman in labor passes some thick meconium as her amniotic fluid ruptures. The client asks the nurse where the baby makes the meconium. What is the correct response by the nurse? a. Fetal intestines b. Fetal kidneys c. Amniotic fluid d. Placenta

ANS: A As the fetus nears term, fetal waste products accumulate in the intestines as dark green-to-black, tarry meconium. Meconium is not produced by the fetal kidneys nor should it be present in the amniotic fluid, which may be an indication of fetal compromise. The placenta does not produce meconium.

With regard to medications, herbs, boosters, and other substances normally encountered by pregnant women, what is important for the nurse to be aware of? a. Both prescription and over-the-counter (OTC) drugs that otherwise are harmless can be made hazardous by metabolic deficiencies of the fetus. b. The greatest danger of drug-caused developmental deficits in the fetus is observed in the final trimester. c. Killed-virus vaccines (e.g., tetanus) should not be administered during pregnancy, but live-virus vaccines (e.g., measles) are permissible. d. No convincing evidence exists that secondhand smoke is potentially dangerous to the fetus.

ANS: A Both prescription and OTC drugs that otherwise are harmless can be made hazardous by metabolic deficiencies of the fetus. This is especially true for new medications and combinations of drugs. The greatest danger of drug-caused developmental defects exists in the interval from fertilization through the first trimester, during which a woman may not realize that she is pregnant. Live-virus vaccines should be part of postpartum care; killed-virus vaccines may be administered during pregnancy. Secondhand smoke is associated with fetal growth restriction and increases in infant mortality.

Which is the most common technique used for the termination of a pregnancy in the second trimester? a. Dilation and evacuation (D&E) b. Methotrexate administration c. Prostaglandin administration d. Vacuum aspiration

ANS: A D&E can be performed at any point up to 20 weeks of gestation. It is more commonly performed between 13 and 16 weeks of gestation. Methotrexate is a cytotoxic drug that causes early abortion by preventing fetal cell division. Prostaglandins are also used for early abortion and work by dilating the cervix and initiating uterine wall contractions. Vacuum aspiration is used for abortions in the first trimester.

8. Which statement indicates that a client requires additional instruction regarding breast self-examination (BSE)? a. "Yellow discharge from my nipple is normal if I'm having my period." b. "I should check my breasts at the same time each month, after my period." c. "I should also feel in my armpit area while performing my breast examination." d. "I should check each breast in a set way, such as in a circular motion."

ANS: A Discharge from the nipples requires further examination from a health care provider. The breasts should be checked at the same time each month. The armpit should also be examined. A circular motion is the best method during which to ascertain any changes in the breast tissue.

Which nutritional recommendation regarding fluids is accurate? a. A woman's daily intake should be eight to ten glasses of water, milk, and/or juice. b. Coffee should be limited to no more than 2 cups, but tea and cocoa can be consumed without worry. c. Of the artificial sweeteners, only aspartame has not been associated with any maternity health concerns. d. Water with fluoride is especially encouraged because it reduces the child's risk of tooth decay.

ANS: A Eight to ten glasses is still the standard for fluids; however, they should be the right fluids. All beverages containing caffeine, including tea, cocoa, and some soft drinks, should be avoided or should be consumed only in limited amounts. Artificial sweeteners, including aspartame, have no ill effects on the normal mother or fetus. However, mothers with phenylketonuria (PKU) should avoid aspartame. Although no evidence indicates that prenatal fluoride consumption reduces childhood tooth decay, fluoride still helps the mother.

A woman is in her seventh month of pregnancy. She reports episodes of nasal congestion and occasional epistaxis. Which statement best describes why this may be happening to this client? a. This respiratory change is normal in pregnancy and caused by an elevated level of estrogen. b. This cardiovascular change is abnormal, and the nosebleeds are an ominous sign. c. The woman is a victim of domestic violence and is being hit in the face by her partner. d. The woman has been likely intranasally using cocaine for several months.

ANS: A Elevated levels of estrogen cause capillaries to become engorged in the respiratory tract, which may result in edema in the nose, larynx, trachea, and bronchi. This congestion may cause nasal stuffiness and epistaxis. Cardiovascular changes in pregnancy may cause edema in the lower extremities. Domestic violence cannot be determined on the basis on the sparse facts provided. If the woman had been hit in the face, then she most likely would have additional physical findings. Cocaine use cannot be determined on the basis on the sparse facts provided.

Which statement regarding emergency contraception is correct? a. Emergency contraception requires that the first dose be taken within 120 hours of unprotected intercourse. b. Emergency contraception may be taken right after ovulation. c. Emergency contraception has an effectiveness rate in preventing pregnancy of approximately 50%. d. Emergency contraception is commonly associated with the side effect of menorrhagia.

ANS: A Emergency contraception should be taken as soon as possible or within 72 hours of unprotected intercourse to prevent pregnancy. If taken before ovulation, follicular development is inhibited, which prevents ovulation. The risk of pregnancy is reduced by as much as 75%. The most common side effect of postcoital contraception is nausea.

A woman asks the nurse, "What protects my baby's umbilical cord from being squashed while the baby's inside of me?" What is the nurse's best response? a. "Your baby's umbilical cord is surrounded by connective tissue called Wharton's jelly, which prevents compression of the blood vessels." b. "Your baby's umbilical cord floats around in blood and amniotic fluid." c. "You don't need to be worrying about things like that." d. "The umbilical cord is a group of blood vessels that are very well protected by the placenta."

ANS: A Explaining the structure and function of the umbilical cord is the most appropriate response. Connective tissue called Wharton's jelly surrounds the umbilical cord, prevents compression of the blood vessels, and ensures continued nourishment of the embryo or fetus. The umbilical cord does not float around in blood or fluid. Telling the client not to worry negates her need for information and discounts her feelings. The placenta does not protect the umbilical cord.

Which term best describes the conscious decision concerning deciding when to conceive or avoid pregnancy as opposed to the intentional prevention of pregnancy during intercourse? a. Family planning b. Birth control c. Contraception d. Assisted reproductive therapy

ANS: A Family planning is the process of deciding when and if to have children. Birth control is the device and/or practice used to reduce the risk of conceiving or bearing children. Contraception is the intentional prevention of pregnancy during sexual intercourse. Assisted reproductive therapy is one of several possible treatments for infertility.

Which guidance might the nurse provide for a client with severe morning sickness? a. Trying lemonade and potato chips b. Drinking plenty of fluids early in the day c. Immediately brushing her teeth after eating d. Never snacking before bedtime

ANS: A Interestingly, some women can tolerate tart or salty foods when they are nauseated. Lemonade and potato chips are an ideal combination. The woman should avoid drinking too much when nausea is most likely, but she should increase her fluid levels later in the day when she feels better. The woman should avoid brushing her teeth immediately after eating. A small snack of cereal and milk or yogurt before bedtime may help the stomach in the morning.

Which statement best describes Kegel exercises? a. Kegel exercises were developed to control or reduce incontinent urine loss. b. Kegel exercises produce a pleasurable vaginal sensation. c. Kegel exercises help manage stress. d. Kegel exercises are ineffective without sufficient calcium in the diet.

ANS: A Kegel exercises help control the urge to urinate. Although these exercises may be pleasurable for some, the most important factor is the control they provide over incontinence. Kegel exercises help manage urination, not stress. Calcium in the diet is important but not related to Kegel exercises.

A pregnant client tells her nurse that she is worried about the blotchy, brownish coloring over her cheeks, nose, and forehead. The nurse can reassure her that this is a normal condition related to hormonal changes. What is the correct term for this integumentary finding? a. Melasma b. Linea nigra c. Striae gravidarum d. Palmar erythema

ANS: A Melasma, (also called chloasma, the mask of pregnancy), usually fades after birth. This hyperpigmentation of the skin is more common in women with a dark complexion. Melasma appears in 50% to 70% of pregnant women. Linea nigra is a pigmented line that runs vertically up the abdomen. Striae gravidarum are also known as stretch marks. Palmar erythema is signified by pinkish red blotches on the hands.

On vaginal examination of a 30-year-old woman, the nurse documents the following findings: profuse, thin, grayish-white vaginal discharge with a "fishy" odor and reports of pruritus. Based upon these findings, which condition would the nurse suspect? a. Bacterial vaginosis b. Candidiasis c. Trichomoniasis d. Gonorrhea

ANS: A Most women with bacterial vaginosis complain of a characteristic "fishy" odor. The discharge is usually profuse, thin, and has a white, gray, or milky color. Some women may also experience mild irritation or pruritus. The discharge associated with candidiasis is thick, white, and lumpy and resembles cottage cheese. Trichomoniasis may be asymptomatic, but women commonly have a characteristic yellow-to-green, frothy, mucopurulent, copious, and malodorous discharge. Women with gonorrhea are often asymptomatic. Although they may have a purulent endocervical discharge, the discharge is usually minimal or absent.

Which nutrient's recommended dietary allowance (RDA) is higher during lactation than during pregnancy? a. Protein b. Iron c. Vitamin A d. Folic acid

ANS: A Nutrient needs for energy—protein, calcium, iodine, zinc, B vitamins, and vitamin C—remain higher during lactation than during pregnancy. The need for iron is not higher during lactation than during pregnancy. A lactating woman does not have a greater requirement for vitamin A than a nonpregnant woman. Folic acid requirements are the highest during the first trimester of pregnancy.

The nurse caring for a pregnant client is evaluating health teaching regarding fetal circulation. Which statement from the client reassures the nurse that the teaching has been effective? a. "Optimal fetal circulation is achieved when I am in the side-lying position." b. "Optimal fetal circulation is achieved when I am on my back with a pillow under my knees." c. "Optimal fetal circulation is achieved when the head of the bed is elevated." d. "Optimal fetal circulation is achieved when I am on my abdomen."

ANS: A Optimal circulation is achieved when the woman is lying at rest on her side. Decreased uterine circulation may lead to intrauterine growth restriction. Previously, it was believed that the left lateral position promoted maternal cardiac output, enhancing blood flow to the fetus. However, it is now known that the side-lying position enhances uteroplacental blood flow. If a woman lies on her back with the pressure of the uterus compressing the vena cava, then blood return to the right atrium is diminished. Although having the head of the bed elevated is recommended and ideal for later in pregnancy, the woman still must maintain a lateral tilt to the pelvis to avoid compressing the vena cava. Many women find lying on their abdomen uncomfortable as pregnancy advances. Side-lying is the ideal position to promote blood flow to the fetus.

After the nurse completes nutritional counseling for a pregnant woman, she asks the client to repeat the instructions to assess the client's understanding. Which statement indicates that the client understands the role of protein in her pregnancy? a. "Protein will help my baby grow." b. "Eating protein will prevent me from becoming anemic." c. "Eating protein will make my baby have strong teeth after he is born." d. "Eating protein will prevent me from being diabetic."

ANS: A Protein is the nutritional element basic to growth. An adequate protein intake is essential to meeting the increasing demands of pregnancy. These demands arise from the rapid growth of the fetus; the enlargement of the uterus, mammary glands, and placenta; the increase in the maternal blood volume; and the formation of the amniotic fluid. Iron intake prevents anemia. Calcium intake is needed for fetal bone and tooth development. Glycemic control is needed in those with diabetes; protein is one nutritional factor to consider for glycemic control but not the primary role of protein intake.

A first-time mother at 18 weeks of gestation is in for her regularly scheduled prenatal visit. The client tells the nurse that she is afraid that she is going into premature labor because she is beginning to have regular contractions. The nurse explains that these are Braxton Hicks contractions. What other information is important for the nurse to share? a. Braxton Hicks contractions should be painless. b. They may increase in frequency with walking. c. These contractions might cause cervical dilation. d. Braxton Hicks contractions will impede oxygen flow to the fetus.

ANS: A Soon after the fourth month of gestation, uterine contractions can be felt through the abdominal wall. Braxton Hicks contractions are regular and painless and continue throughout the pregnancy. Although they are not painful, some women complain that they are annoying. This type of contraction usually ceases with walking or exercise. Braxton Hicks contractions can be mistaken for true labor; however, they do not increase in intensity, frequency, or cause cervical dilation. These contractions facilitate uterine blood flow through the intervillous spaces of the placenta and thereby promote oxygen delivery to the fetus.

Which infection control practice should the nurse use when providing eye prophylaxis to a term newborn possibly infected with human immunodeficiency virus (HIV)? a. Wearing gloves. b. Wearing mouth, nose, and eye protection. c. Wearing a mask. d. Washing the hands after medication administration.

ANS: A Standard Precautions should be consistently used in the care of all persons. Personal protective equipment in the form of gloves should be worn during infant eye prophylaxis, care of the umbilical cord, circumcision site care, diaper changes, handling of colostrum, and parenteral procedures. Masks are worn during respiratory isolation or if the health care practitioner has a cough. Mouth, eye, and nose protection are used to protect the mucous membranes if client-care activities are likely to generate splashes or sprays of body fluids. The hands should be washed both before having contact with the client and after administering medications.

Why might it be more difficult to diagnose appendicitis during pregnancy? a. The appendix is displaced upward and laterally, high and to the right. b. The appendix is displaced upward and laterally, high and to the left. c. The appendix is deep at the McBurney's point. d. The appendix is displaced downward and laterally, low and to the right.

ANS: A The appendix is displaced high and to the right, not to the left. It is displaced beyond the McBurney's point and is not displaced in a downward direction.

A woman has chosen the calendar method of conception control. Which is the most important action the nurse should perform during the assessment process in preparation to discuss the implementation of this method? a. Obtain a history of the woman's menstrual cycle lengths for the past 6 to 12 months. b. Determine the client's weight gain and loss pattern for the previous year. c. Examine skin pigmentation and hair texture for hormonal changes. d. Explore the client's previous experiences with conception control.

ANS: A The calendar method of conception control is based on the number of days in each cycle, counting from the first day of menses. The fertile period is determined after the lengths of menstrual cycles have been accurately recorded for 6 months. Weight gain or loss may be partly related to hormonal fluctuations, but it has no bearing on the use of the calendar method. Integumentary changes may be related to hormonal changes, but they are not indicators for use of the calendar method. Exploring previous experiences with conception control may demonstrate client understanding and compliancy, but these experiences are not the most important aspect to assess for the discussion of the calendar method.

Ovarian function and hormone production decline during which transitional phase? a. Climacteric b. Menarche c. Menopause d. Puberty

ANS: A The climacteric phase is a transitional period during which ovarian function and hormone production decline. Menarche is the term that denotes the first menstruation. Menopause refers only to the last menstrual period. Puberty is a broad term that denotes the entire transitional period between childhood and sexual maturity.

Which behavior indicates that a woman is "seeking safe passage" for herself and her infant? a. She keeps all prenatal appointments. b. She "eats for two." c. She slowly drives her car. d. She wears only low-heeled shoes.

ANS: A The goal of prenatal care is to foster a safe birth for the infant and mother. Although properly eating, carefully driving, and using proper body mechanics all are healthy measures that a mother can take, obtaining prenatal care is the optimal method for providing safety for both herself and her baby.

The various systems and organs of the fetus develop at different stages. Which statement is most accurate? a. Cardiovascular system is the first organ system to function in the developing human. b. Hematopoiesis originating in the yolk sac begins in the liver at 10 weeks of gestation. c. Body changes from straight to C-shape occurs at 8 weeks of gestation. d. Gastrointestinal system is mature at 32 weeks of gestation.

ANS: A The heart is developmentally complete by the end of the embryonic stage. Hematopoiesis begins in the liver during the sixth week. The body becomes C-shaped at 21 weeks of gestation. The gastrointestinal system is complete at 36 weeks of gestation.

Which viral sexually transmitted infection is characterized by a primary infection followed by recurrent episodes? a. Herpes simplex virus 2 (HSV-2) b. Human papillomavirus (HPV) c. Human immunodeficiency virus (HIV) d. Cytomegalovirus (CMV)

ANS: A The initial HSV genital infection is characterized by multiple painful lesions, fever, chills, malaise, and severe dysuria; it may last 2 to 3 weeks. Recurrent episodes of the HSV infection commonly have only local symptoms that usually are less severe than those of the initial infection. With HPV infection, lesions are a chronic problem. The HIV is a retrovirus. Seroconversion to HIV positivity usually occurs within 6 to 12 weeks after the virus has entered the body. Severe depression of the cellular immune system associated with the HIV infection characterizes AIDS, which has no cure. In most adults, the onset of CMV infection is uncertain and asymptomatic. However, the disease may become a chronic, persistent infection.

When the nurse is alone with a battered client, the client seems extremely anxious and says, "It was all my fault. The house was so messy when he got home, and I know he hates that." What is the most suitable response by the nurse? a. "No one deserves to be hurt. It's not your fault. How can I help you?" b. "What else do you do that makes him angry enough to hurt you?" c. "He will never find out what we talk about. Don't worry. We're here to help you." d. "You have to remember that he is frustrated and angry, so he takes it out on you."

ANS: A The nurse should stress that the client is not at fault. Asking what the client did to make her husband angry is placing the blame on the woman and would be an inappropriate statement. The nurse should not provide false reassurance. To assist the woman, the nurse should be honest. Often the batterer will find out about the conversation.

What is the correct term used to describe the mucous plug that forms in the endocervical canal? a. Operculum b. Leukorrhea c. Funic souffle d. Ballottement

ANS: A The operculum protects against bacterial invasion. Leukorrhea is the mucus that forms the endocervical plug (the operculum). The funic souffle is the sound of blood flowing through the umbilical vessels. Ballottement is a technique for palpating the fetus.

A client at 34 weeks of gestation seeks guidance from the nurse regarding personal hygiene. Which information should the nurse provide? a. Tub bathing is permitted even in late pregnancy unless membranes have ruptured. b. The perineum should be wiped from back to front. c. Bubble bath and bath oils are permissible because they add an extra soothing and cleansing action to the bath. d. Expectant mothers should use specially treated soap to cleanse the nipples.

ANS: A The primary danger from taking baths is falling in the tub. The perineum should be wiped from front to back. Bubble baths and bath oils should be avoided because they may irritate the urethra. Soap, alcohol, ointments, and tinctures should not be used to cleanse the nipples because they remove protective oils.

Which female reproductive organ is responsible for cyclic menstruation? a. Uterus b. Ovary c. Vaginal vestibule d. Urethra

ANS: A The uterus is responsible for cyclic menstruation while also housing and nourishing the fertilized ovum and the fetus. The ovaries are responsible for ovulation and the production of estrogen. The vaginal vestibule is an external organ that has openings to the urethra and vagina. The urethra is not a reproductive organ, although it is found in the area.

Which statement made by a nurse regarding the prophylactic vaccination to prevent human papillomavirus (HPV) demonstrates a need for further education? a. "Currently there is only one vaccine for the HPV available." b. "The vaccine is given in three doses over a 6-month period." c. "The vaccine is recommended for both boys and girls." d. "Ideally, the vaccine is administered before the first sexual contact."

ANS: A Three vaccines for HPV are available—Cervarix, Gardasil and Gardasil 9—and other vaccines continue to be investigated. They are most effective if administered before the first sexual contact. Recommendations are that vaccines be administered to 11- and 12-year-old girls and boys. The vaccine can be given to girls as young as 9 years of age and young women ages 13 to 26 years in three doses over a 6-month period.

Which signs and symptoms should a woman immediately report to her health care provider? (Select all that apply.) a. Vaginal bleeding b. Rupture of membranes c. Heartburn accompanied by severe d. headache e. Decreased libido f. Urinary frequency

ANS: A, B, C Vaginal bleeding, rupture of membranes, and severe headaches are signs of potential complications in pregnancy. Clients should be advised to report these signs to their health care provider. Decreased libido and urinary frequency are common discomforts of pregnancy that do not require immediate health care interventions.

Most women with uncomplicated pregnancies can use the nurse as their primary source for nutritional information. However, the nurse or midwife may need to refer a client to a registered dietitian for in-depth nutritional counseling. Which conditions would require such a consultation? (Select all that apply.) a. Preexisting or gestational illness such as diabetes b. Ethnic or cultural food patterns c. Obesity d. Vegetarian diets e. Multifetal pregnancy

ANS: A, B, C, D The nurse should be especially aware that conditions such as diabetes can require in-depth dietary planning and evaluation. To prevent issues with hypoglycemia and hyperglycemia, as well as an increased risk for perinatal morbidity and mortality, the client with a preexisting or gestational illness would benefit from a referral to a dietitian. Consultation with a dietitian may ensure that cultural food beliefs are congruent with modern knowledge of fetal development and that adjustments can be made to ensure that all nutritional needs are met. The obese pregnant client may be under the misapprehension that, because of her excess weight, little or no weight gain is necessary. According to the Institute of Medicine, a client with a BMI in the obese range should gain at least 7 kg to ensure a healthy outcome. This client may require in-depth counseling on the optimal food choices. The vegetarian client needs to have her dietary intake carefully assessed to ensure that the optimal combination of amino acids and protein intake is achieved. Very strict vegetarians (vegans) who consume only plant products may also require vitamin B and mineral supplementation. A multifetal pregnancy can be managed by increasing the number of servings of complex carbohydrates and proteins.

Many factors, male and female, contribute to normal fertility. Which factors are possible causes for female infertility? (Select all that apply.) a. Congenital or developmental b. Hormonal or ovulatory c. Tubal or peritoneal d. Uterine e. Emotional or psychologic

ANS: A, B, C, D Female infertility can be attributed to alterations in any one of these systems along with possible vaginal-cervical factors. Although the diagnosis and treatment of infertility require considerable emotional investment and may cause psychologic stress, considered factors associated with infertility. Feelings connected with infertility are many and complex. Resolve is an organization that provides support, advocacy, and education for both clients and health care providers.

A woman who has undergone an induced abortion be instructed to return to the emergency department when what situation exists? (Select all that apply.) a. Fever higher than 38 C b. Chills c. Foul-smelling vaginal discharge d. Bleeding greater than 2 pads in 2 hours e. Abdominal tenderness and or pain

ANS: A, B, C, D, E The client should report to a health care facility for any of the following symptoms: fever higher than 38 C, chills, bleeding more than two saturated pads in 2 hours or heavy bleeding lasting for days, foul-smelling discharge, abdominal tenderness or pain, and cramping or backache.

Pregnancy is a hypercoagulable state in which women are at a fivefold to sixfold increased risk for thromboembolic disease. The tendency for blood to clot is greater, attributable to an increase in various clotting factors. Which of these come into play during pregnancy? (Select all that apply.) a. Factor VII b. Factor VIII c. Factor IX d. Factor XIII e. Fibrinogen

ANS: A, B, C, E Factors VII, VIII, IX, X, and fibrinogen increase in pregnancy. Factors that inhibit coagulation decrease. Fibrinolytic activity (dissolving of a clot) is depressed during pregnancy and the early postpartum period to protect the women from postpartum hemorrhage.

Counseling and education are critical components of the nursing care of women with herpes infections. Clients should be taught to identify triggers that might result in a herpes attack. Which factors are possible triggers for a recurrence? (Select all that apply.) a. Menstruation b. Trauma c. Febrile illness d. Soap e. Ultraviolet light

ANS: A, B, C, E Stress, menstruation, trauma, febrile illness, chronic illnesses, and ultraviolet light have all been found to trigger genital herpes. Women might elect to keep a diary of symptoms to help identify stressors. Lesions should be cleansed with saline or simple soap and water. Lesions can be kept dry by using a blow dryer, wearing cotton underwear, and wearing loose clothing. Tea bags or hydrogen peroxide might also be helpful.

Many pregnant teenagers wait until the second or third trimester to seek prenatal care. What should the nurse recognize as reasons for this delay? (Select all that apply.) a. Lack of realization that they are pregnant b. Uncertainty as to where to go for care c. Continuing to deny the pregnancy d. Desire to gain control over their situation e. Wanting to hide the pregnancy as long as possible

ANS: A, B, C, E These reasons are all valid explanations why teens delay seeking prenatal care. An adolescent often has little to no understanding of the increased physiologic needs that a pregnancy places on her body. Once care is sought, it is often sporadic, and many appointments are usually missed. The nurse should formulate a diagnosis that assists the pregnant teen to receive adequate prenatal care. Planning for her pregnancy and impending birth actually provides some sense of control for the teen and increases her feelings of competency. Receiving praise from the nurse when she attends her prenatal appointments will reinforce the teen's positive self-image.

Which infections, collaboratively referred to as TORCH infections, are capable of crossing the placenta and causing serious prenatal effects on the fetus? (Select all that apply.) a. Toxoplasmosis b. Cytomegalovirus (CMV) c. Rubella d. Clostridium e. Herpes simplex 1 (HSV-1)

ANS: A, B, C, E Toxoplasmosis, cytomegalovirus (CMV), rubella virus, and HSV are collectively known as TORCH infections. Clostridium is not associated Torch infections.

The nurse, responsible for providing health teaching regarding the side effects of combined oral contraceptives (COCs), should discuss what issues? (Select all that apply.) a. Gallbladder disease b. Myocardial infarction c. Hypotension d. Breast tenderness e. Dry skin and scalp

ANS: A, B, D Serious side effects include stroke, myocardial infarction, hypertension, gallbladder disease, and liver tumors. More common side effects include nausea, breast tenderness, fluid retention, increased appetite, and oily skin and scalp

The nurse understands that the risk of perinatal transmission of human immunodeficiency virus (HIV) can be significantly decreased by a number of prophylactic interventions. Which interventions should be included in the plan of care of a pregnant client with a viral load of more than 1000 copies/ml.? (Select all that apply.) a. Intrapartum treatment with antiviral medications b. Cesarean birth c. Postpartum treatment with antiviral medications d. Avoidance of breastfeeding e. Pneumococcal, HBV, and Haemophilus influenzae vaccine

ANS: A, B, D The prophylactic measures of prenatal antiviral use, elective cesarean birth, and formula feeding reduce the transmission of the HIV to as low as 1% to 2%. The client who refuses a cesarean birth should be given intravenous antiviral therapy during labor. Ideally, medications should be given prenatally. Administration of antiviral drugs in the postpartum period will not reduce transmission to the infant. All women who are HIV positive should be encouraged to receive these immunizations. They will not reduce the risk of perinatal transmission.

Which statements regarding menstruation (periodic uterine bleeding) are accurate? (Select all that apply.) a. Menstruation occurs every 28 days. b. During menstruation, the entire uterine lining is shed. c. Menstruation begins 7 to 10 days after ovulation. d. Menstruation leads to fertilization. e. Average blood loss during menstruation is 50 ml.

ANS: A, B, E Menstruation is the periodic uterine bleeding that is controlled by a feedback system involving three cycles: the endometrial cycle, the hypothalamic-pituitary cycle, and the ovarian cycle. The average length of a menstrual cycle is 28 days; however, variations are normal. During the endometrial cycle, the functional two thirds of the endometrium is shed. The average blood loss is 50 ml with a normal range of 20 to 80 ml. Menstruation occurs 14 days after ovulation. The lack of fertilization leads to menstruation.

A woman has just moved to the United States from Mexico. She is 3 months pregnant and has arrived for her first prenatal visit. During her assessment interview, the nurse learns that the client has not had any immunizations. Which immunizations should she receive at this point in her pregnancy? (Select all that apply.) a. Tetanus b. Diphtheria c. Chickenpox d. Rubella e. Hepatitis B

ANS: A, B, E Vaccines consisting of killed viruses may be used. Those that may be administered during pregnancy include tetanus, diphtheria, recombinant hepatitis B, and rabies vaccines. Immunizations with live or attenuated viruses are contraindicated during pregnancy because of their potential teratogenicity. Live-virus vaccines include those for measles (rubeola and rubella), chickenpox, and mumps.

The diagnosis of pregnancy is based on which positive signs of pregnancy? (Select all that apply.) a. Identification of fetal heartbeat b. Palpation of fetal outline c. Visualization of the fetus d. Verification of fetal movement e. Positive hCG test

ANS: A, C, D Identification of a fetal heartbeat, the visualization of the fetus, and verification of fetal movement are all positive, objective signs of pregnancy. Palpation of fetal outline and positive hCG test are probable signs of pregnancy. A tumor also can be palpated. Medication and tumors may lead to false-positive results on pregnancy tests.

Relating to the fetal circulatory system, which special characteristics allow the fetus to obtain enough oxygen from the maternal blood? (Select all that apply.) a. Fetal hemoglobin (Hb) carries 20% to 30% more oxygen than maternal Hb. b. Fetal Hb carries 40% to 50% more oxygen than maternal Hb. c. Hb concentration is 50% higher than that of the mother. d. Fetal heart rate is 110 to 160 beats per minute. e. Fetal heart rate is 160 to 200 beats per minute.

ANS: A, C, D The following three special characteristics enable the fetus to obtain sufficient oxygen from maternal blood: (1) the fetal Hb carries 20% to 30% more oxygen; (2) the concentration is 50% higher than that of the mother; and (3) the fetal heart rate is 110 to 160 beats per minute, a cardiac output that is higher than that of an adult.

A probable cause for increasing infertility is the societal delay in pregnancy until later in life. What are the natural reasons for the decrease in female fertility? (Select all that apply.) a. Ovulation dysfunction b. Endocrine dysfunction c. Organ damage from toxins d. Endometriosis e. Tubal infections

ANS: A, C, D, E All of these factors may result in a cumulative effect, decreasing fertility in women. Male infertility is more often caused by unfavorable sperm production attributable to endocrine dysfunction or cumulative metabolic disease.

The nurse is reviewing the educational packet provided to a client about tubal ligation. Which information regarding this procedure is important for the nurse to share? (Select all that apply.) a. "It is highly unlikely that you will become pregnant after the procedure." b. "Tubal ligation is an effective form of 100% permanent sterilization. You won't be able to get pregnant." c. "Sterilization offers some form of protection against sexually transmitted infections (STIs)." d. "Sterilization offers no protection against sexually transmitted infections (STIs)." e. "Your menstrual cycle will greatly increase after your sterilization."

ANS: A, D A woman is unlikely to become pregnant after tubal ligation. However, sterilization offers no protection against STIs and is not 100% effective. Typically, the menstrual cycle remains the same after a tubal ligation.

During the first trimester, which of the following changes regarding her sexual drive should a client be taught to expect? a. Increased sexual drive, because of enlarging breasts b. Decreased sexual drive, because of nausea and fatigue c. No change in her sexual drive d. Increased sexual drive, because of increased levels of female hormones

ANS: B A pregnant woman usually experiences a decrease, not an increase, in libido during the first trimester. Maternal physiologic changes, such as breast enlargement, nausea, fatigue, abdominal changes, perineal enlargement, leukorrhea, pelvic vasocongestion, and orgasmic responses, may affect sexuality and sexual expression. Libido may be depressed in the first trimester but often increases during the second and third trimesters. During pregnancy, the breasts may become enlarged and tender, which tends to interfere with coitus, thereby decreasing the desire to engage in sexual activity.

Which action is the highest priority for the nurse when educating a pregnant adolescent? a. Emphasize the need to eliminate common teenage snack foods because they are high in fat and sodium. b. Determine the weight gain needed to meet adolescent growth and add 35 pounds. c. Suggest that she not eat at fast-food restaurants to avoid foods of poor nutritional value. d. Realize that most adolescents are unwilling to make dietary changes during pregnancy.

ANS: B Adolescents should gain in the upper range of the recommended weight gain. They also need to gain weight that would be expected for their own normal growth. Changes in the diet should be kept at a minimum. Snack foods can be included in moderation, and other foods can be added to make up for lost nutrients. Eliminating fast foods would make the adolescent appear different to her peers. The client should be taught to choose foods that add needed nutrients. Adolescents are willing to make changes; however, they still have the need to be similar to their peers.

Many clients are concerned about the increased levels of mercury in fish and may be reluctant to include this source of nutrients in their diet. What is the best advice for the nurse to provide? a. "Canned white tuna is a preferred choice for inclusion in your diet." b. "Shark, swordfish, and mackerel are types of fish that have high mercury levels" c. "Fish caught in local waterways is the safest to consume." d. "Avoid salmon and shrimp since they contain high levels of mercury."

ANS: B As a precaution, the pregnant client should avoid eating shark, swordfish, and mackerel, as well as the less common tilefish. High levels of mercury can harm the developing nervous system of the fetus. Assisting the client in understanding the differences between numerous sources of mercury is essential for the nurse. A pregnant client may eat as much as 12 ounces a week of canned light tuna; however, canned white, albacore, or tuna steaks contain higher levels of mercury and should be limited to no more than 6 ounces per week. Pregnant women and mothers of young children should check with local advisories about the safety of fish caught by families and friends in nearby bodies of water. If no information is available, then these fish sources should be avoided, limited to less than 6 ounces per week, or the only fish consumed that week. Commercially caught fish that is low in mercury includes salmon, shrimp, pollock, or catfish. The pregnant client may eat up to 12 ounces of commercially caught fish per week. Additional information on levels of mercury in commercially caught fish is available at www.cfsan.fda.gov.

What type of cultural concern is the most likely deterrent to many women seeking prenatal care? a. Religion b. Modesty c. Ignorance d. Belief that physicians are evil

ANS: B A concern for modesty is a deterrent to many women seeking prenatal care. For some women, exposing body parts, especially to a man, is considered a major violation of their modesty. Many cultural variations are found in prenatal care. Even if the prenatal care described is familiar to a woman, some practices may conflict with the beliefs and practices of a subculture group to which she belongs.

A client states that she plans to breastfeed her newborn infant. What guidance would be useful for this new mother? a. The mother's intake of vitamin C, zinc, and protein can now be lower than during pregnancy. b. Caffeine consumed by the mother accumulates in the infant, who may be unusually active and wakeful. c. Critical iron and folic acid levels must be maintained. d. Lactating women can go back to their prepregnant caloric intake.

ANS: B A lactating woman needs to avoid consuming too much caffeine. Vitamin C, zinc, and protein levels need to be moderately higher during lactation than during pregnancy. The recommendations for iron and folic acid are lower during lactation. Lactating women should consume approximately 500 kcal more than their prepregnancy intake, at least 1800 kcal daily overall.

Assessment of a woman's nutritional status includes a diet history, medication regimen, physical examination, and relevant laboratory tests. Which finding might require consultation to a higher level of care? a. Oral contraceptive use may interfere with the absorption of iron. b. Illnesses that have created nutritional deficits, such as PKU, may require nutritional care before conception. c. The woman's socioeconomic status and educational level are not relevant to her examination; they are the province of the social worker. d. Testing for diabetes is the only nutrition-related laboratory test most pregnant women need.

ANS: B A registered dietitian can help with therapeutic diets. Oral contraceptive use may interfere with the absorption of folic acid. Iron deficiency can appear if placement of an intrauterine device (IUD) results in blood loss. A woman's finances can affect her access to good nutrition; her education (or lack thereof) can influence the nurse's teaching decisions. The nutrition-related laboratory test that pregnant women usually need is a screen for anemia.

A woman who is 16 weeks pregnant asks the nurse, "Is it possible to tell by ultrasound if the baby is a boy or girl yet?" What is the best answer? a. "A baby's sex is determined as soon as conception occurs." b. "The baby has developed enough to enable us to determine the sex by examining the genitals through an ultrasound scan." c. "Boys and girls look alike until approximately 20 weeks after conception, and then they begin to look different." d. "It might be possible to determine your baby's sex, but the external organs look very similar right now."

ANS: B Although gender is determined at conception, the external genitalia of males and females look similar through the ninth week. By the twelfth week, the external genitalia are distinguishable as male or female.

Which sign of a potential complication is the most important for the nurse to share with the client? a. Constipation b. Alteration in the pattern of fetal movement c. Heart palpitations d. Edema in the ankles and feet at the end of the day

ANS: B An alteration in the pattern or amount of fetal movement may indicate fetal jeopardy. Constipation, heart palpitations, and ankle and foot edema are normal discomforts of pregnancy that occur in the second and third trimesters.

The musculoskeletal system adapts to the changes that occur throughout the pregnancy. Which musculoskeletal alteration should the client expect? a. Her center of gravity will shift backward. b. She will have increased lordosis. c. She will have increased abdominal muscle tone. d. She will notice decreased mobility of her pelvic joints.

ANS: B An increase in the normal lumbosacral curve (lordosis) develops, and a compensatory curvature in the cervicodorsal region develops to help her maintain balance. The center of gravity shifts forward. She will have decreased abdominal muscle tone and will notice increased mobility of her pelvic joints.

Which statement regarding the structure and function of the placenta is correct? a. Produces nutrients for fetal nutrition b. Secretes both estrogen and progesterone c. Forms a protective, impenetrable barrier to microorganisms such as bacteria and viruses d. Excretes prolactin and insulin

ANS: B As one of its early functions, the placenta acts as an endocrine gland, producing four hormones necessary to maintain the pregnancy and to support the embryo or fetus: human chorionic gonadotropin (hCG), human placental lactogen (hPL), estrogen, and progesterone. The placenta does not produce nutrients. It functions as a means of metabolic exchange between the maternal and fetal blood supplies. Many bacteria and viruses can cross the placental membrane.

The Centers for Disease Control and Prevention (CDC) recommends which therapy for the treatment of the HPV? a. Miconazole ointment b. Topical podofilox 0.5% solution or gel c. Two doses of penicillin administered intramuscularly (IM) d. Metronidazole by mouth

ANS: B Available treatments are imiquimod, podophyllin, and podofilox. Miconazole ointment is used to treat athlete's foot. Penicillin IM is used to treat syphilis. Metronidazole is used to treat bacterial vaginosis.

Which contraceptive method should the nurse identify as protecting against sexually transmitted infections (STIs) and the human immunodeficiency virus (HIV)? a. Periodic abstinence b. Barrier methods c. Hormonal methods d. Same protection with all methods

ANS: B Barrier methods, such as condoms, protect against STIs and the HIV the best of all contraceptive methods. Periodic abstinence and hormonal methods, such as birth control pills, offer no protection against STIs or the HIV.

A woman who has a seizure disorder and takes barbiturates and phenytoin sodium daily asks the nurse about the pill as a contraceptive choice. What is the nurse's best response? a. "Oral contraceptives are a highly effective method, but they have some side effects." b. "Your current medications will reduce the effectiveness of the pill." c. "Oral contraceptives will reduce the effectiveness of your seizure medication." d. "The pill is a good choice for a woman of your age and with your personal history."

ANS: B Because the liver metabolizes oral contraceptives, their effectiveness is reduced when they are simultaneously taken with anticonvulsants. Stating that the pill is an effective birth control method with side effects is a true statement, but this response is not the most appropriate. The anticonvulsant reduces the effectiveness of the pill, not the other way around. Stating that the pill is a good choice for a woman of her age and personal history does not teach the client that the effectiveness of the pill may be reduced because of her anticonvulsant therapy.

16. Which renal system adaptation is an anticipated anatomic change of pregnancy? a. Increased urinary output makes pregnant women less susceptible to urinary infections. b. Increased bladder sensitivity and then compression of the bladder by the enlarging uterus result in the urge to urinate even when the bladder is almost empty. c. Renal (kidney) function is more efficient when the woman assumes a supine position. d. Using diuretic agents during pregnancy can help keep kidney function regular.

ANS: B Bladder sensitivity and then compression of the bladder by the uterus result in the urge to urinate more often, even when the bladder is almost empty. A number of anatomic changes in pregnancy make a woman more susceptible to urinary tract infections. Renal function is more efficient when the woman lies in the lateral recumbent position and is less efficient when she is supine. Diuretic use during pregnancy can overstress the system and cause problems.

A 3-year-old girl's mother is 6 months pregnant. What concern is this child most likely to verbalize? a. How the baby will get out? b. How will the baby eat? c. Will you die having the baby? d. What color eyes will the baby have?

ANS: B By age 3 or 4 years, children like to be told the story of their own beginning and accept it being compared with the present pregnancy. They like to listen to the fetal heartbeat and feel the baby move. Sometimes they worry about how the baby is being fed and what it will wear. School-age children take a more clinical interest in their mother's pregnancy and may want to know "How did the baby get in there?" and "How will it get out?" Whether the child's mother will die does not tend to be the focus of her questions about the impending birth of a sibling. The baby's eye color does not tend to be the focus of children's questions about the impending birth of a sibling.

Which statement regarding the various forms of hepatitis is accurate? a. Vaccine exists for hepatitis C virus (HCV) but not for hepatitis B (HBV). b. Hepatitis A (HAV) is acquired by eating contaminated food or drinking polluted water. c. Hepatitis B (HBV) is less contagious than HIV. d. Incidences of hepatitis C (HCV) is decreasing.

ANS: B Contaminated milk and shellfish are common sources of infection for HAV. A vaccine exists for HBV but not for HCV. HBV is more contagious than HIV. The incidence of HCV is on the rise.

Which action should the nurse first take when meeting with a new client to discuss contraception? a. Obtain data about the frequency of coitus. b. Determine the woman's level of knowledge concerning contraception.. c. Assess the woman's willingness to touch her genitals and cervical mucus. d. Evaluate the woman's contraceptive life plan.

ANS: B Determining the woman's level of knowledge concerning contraception and her commitment to any particular method is the primary step of this nursing assessment and necessary before completing the process and moving on to a nursing diagnosis. Once the client's level of knowledge is determined, the nurse can interact with the woman to compare options, reliability, cost, comfort level, protection from STIs, and her partner's willingness to participate. Although important, obtaining data about the frequency of coitus is not the first action that the nurse should undertake when completing an assessment. Data should include not only the frequency of coitus but also the number of sexual partners, level of contraceptive involvement, and the partner's objections. Assessing the woman's willingness to touch herself is a key factor for the nurse to discuss should the client express an interest in using one of the fertility awareness methods of contraception. The nurse must be aware of the client's plan regarding whether she is attempting to prevent conception, delay conception, or conceive.

During a health history interview, a woman states that she thinks that she has "bumps" on her labia. She also states that she is not sure how to check herself. The correct response by the nurse would be what? a. Reassure the woman that the examination will reveal any problems. b. Explain the process of vulvar self-examination and reassure the woman that she should become familiar with normal and abnormal findings during the examination. c. Reassure the woman that "bumps" can be treated. d. Reassure her that most women have "bumps" on their labia.

ANS: B During the assessment and evaluation, the responsibility for self-care, health promotion, and enhancement of wellness is emphasized. The pelvic examination provides a good opportunity for the practitioner to emphasize the need for regular vulvar self-examination. Providing reassurance to the woman concerning the "bumps" would not be an accurate response.

Which trait is least likely to be displayed by a woman experiencing intimate partner violence (IPV)? a. Social isolation b. Assertive personality c. Reoccurring depression d. Dependent personality

ANS: B Every segment of society is represented among women who are suffering abuse. However, traits of assertiveness, independence, and willingness to take a stand have been documented as more characteristic of women who are in nonviolent relationships. Women who are financially more dependent have fewer resources and support systems, exhibit symptoms of depression, and are more often seen as victims.

Which phase does not belong in Lenore Walker's three-cycle pattern of violence? a. Tension-building state b. Frustration, followed by violence c. Acute battering incident d. Kindness and contrite, loving behavior

ANS: B Frustration, followed by violence, is not part of the cycle of violence. The tension-building state is also known as phase I of the cycle. The batterer expresses dissatisfaction and hostility with violent outbursts. The woman senses anger and anxiously tries to placate him. An acute battering incident is phase II of the cycle. It results in the man's uncontrollable discharge of tension toward the woman. Outbursts can last from several hours to several days and may involve kicking, punching, slapping, choking, burns, broken bones, and the use of weapons. Phase III of the cycle is sometimes referred to as the honeymoon, kindness and contrite, and loving behavior phase, during which the batterer feels remorseful and profusely apologizes. He tries to help the woman and often showers her with gifts.

Which statement concerning neurologic and sensory development in the fetus is correct? a. Brain waves have been recorded on an electroencephalogram as early as the end of the first trimester (12 weeks of gestation). b. Fetuses respond to sound by 24 weeks of gestation and can be soothed by the sound of the mother's voice. c. Eyes are first receptive to light at 34 to 36 weeks of gestation. d. At term, the fetal brain is at least one third the size of an adult brain.

ANS: B Hearing develops early and is fully developed at birth. Brain waves have been recorded at week 8. Eyes are receptive to light at 28 weeks of gestation. The fetal brain is approximately one fourth the size of an adult brain.

A client is seen at the clinic at 14 weeks of gestation for a follow-up appointment. At which level does the nurse expect to palpate the fundus? a. Nonpalpable above the symphysis at 14 weeks of gestation. b. Slightly above the symphysis pubis c. At the level of the umbilicus d. Slightly above the umbilicus

ANS: B In normal pregnancies, the uterus grows at a predictable rate. It may be palpated above the symphysis pubis sometime between the 12th and 14th weeks of pregnancy. As the uterus grows, it may be palpated above the symphysis pubis sometime between the 12th and 14th weeks of pregnancy. At 14 weeks, the uterus is not yet at the level of the umbilicus. The fundus is not palpable above the umbilicus until 22 to 24 weeks of gestation.

The pancreas forms in the foregut during the 5th to 8th week of gestation. A client with poorly controlled gestational diabetes asks the nurse what the effects of her condition will be on the fetus. What is the best response by the nurse? Poorly controlled maternal gestational diabetes will: a. produce fetal hypoglycemia. b. result in a macrocosmic fetus. c. result in a microcosmic fetus. d. enhance lung maturation.

ANS: B Insulin is produced by week 20 of gestation. In the fetus of a mother with uncontrolled diabetes, maternal hypoglycemia produces fetal hypoglycemia and macrocosmia results. Hyperinsulinemia blocks lung maturation, placing the neonate at risk for respiratory distress.

To prevent gastrointestinal (GI) upset, when should a pregnant client be instructed to take the recommended iron supplements? a. On a full stomach b. At bedtime c. After eating a meal d. With milk

ANS: B Iron supplements taken at bedtime may reduce GI upset and should be taken at bedtime if abdominal discomfort occurs when iron supplements are taken between meals. Iron supplements are best absorbed if they are taken when the stomach is empty. Bran, tea, coffee, milk, and eggs may reduce absorption.

The nurse knows that teaching about the natural family planning method of contraception was effective when the couple responds that an ovum is considered fertile for which period of time? a. 6 to 8 hours b. 24 hours c. 2 to 3 days d. 1 week

ANS: B Most ova remain fertile for approximately 24 hours after ovulation, much longer than 6 to 8 hours. However, ova do not remain fertile for 2 to 3 days or are viable for 1 week. If unfertilized by a sperm after 24 hours, the ovum degenerates and is reabsorbed.

A patient in her first trimester complains of nausea and vomiting. She asks, "Why does this happen?" What is the nurse's best response? a. "Nausea and vomiting are due to an increase in gastric motility." b. "Nausea and vomiting may be due to changes in hormones." c. "Nausea and vomiting are related to an increase in glucose levels." d. "Nausea and vomiting are caused by a decrease in gastric secretions."

ANS: B Nausea and vomiting are believed to be caused by increased levels of hormones, decreased gastric motility, and hypoglycemia. Gastric motility decreases during pregnancy. Glucose levels decrease in the first trimester. Although gastric secretions decrease, these secretions are not the primary cause of the nausea and vomiting.

A male client asks the nurse why it is better to purchase condoms that are not lubricated with nonoxynol-9 (a common spermicide). Which response by the nurse is the most accurate? a. "The lubricant prevents vaginal irritation." b. "Nonoxynol-9 does not provide protection against STIs as originally thought" c. "The additional lubrication improves sex." d. "Nonoxynol-9 improves penile sensitivity."

ANS: B Nonoxynol-9 does not provide protection against STIs as originally thought; it has also been linked to an increase in the transmission of the HIV and can cause genital lesions. Nonoxynol-9 may cause vaginal irritation, has no effect on the quality of sexual activity, and has no effect on penile sensitivity.

Maternal nutritional status is an especially significant factor of the many that influence the outcome of pregnancy. Why is this the case? a. Maternal nutritional status is extremely difficult to adjust because of an individual's ingrained eating habits. b. Adequate nutrition is an important preventative measure for a variety of problems. c. Women love obsessing about their weight and diets. d. A woman's preconception weight becomes irrelevant.

ANS: B Nutritional status draws so much attention not only for its effect on a healthy pregnancy and birth but also because significant changes are within relatively easy reach. Pregnancy is a time when many women are motivated to learn about adequate nutrition and make changes to their diet that will benefit their baby. Pregnancy is not the time to begin a weight loss diet. Clients and their caregivers should still be concerned with appropriate weight gain.

What fatty acids (classified as hormones) are found in many body tissues with complex roles in many reproductive functions? a. GnRH b. Prostaglandins (PGs) c. Follicle-stimulating hormone (FSH) d. Luteinizing hormone (LH)

ANS: B PGs affect smooth muscle contraction and changes in the cervix. GnRH is part of the hypothalamic-pituitary cycle, which responds to the rise and fall of estrogen and progesterone. FSH is part of the hypothalamic-pituitary cycle, which responds to the rise and fall of estrogen and progesterone. LH is part of the hypothalamic-pituitary cycle, which responds to the rise and fall of estrogen and progesterone.

A pregnant woman at 18 weeks of gestation calls the clinic to report that she has been experiencing occasional backaches of mild-to-moderate intensity. Which intervention should the nurse recommend? a. Kegel exercises b. Pelvic rock exercises c. Softer mattress d. Bed rest for 24 hours

ANS: B Pelvic rock exercises may help stretch and strengthen the abdominal and lower back muscles and relieve low back pain. Stretching and other exercises to relieve back pain should be performed several times a day. Kegel exercises increase the tone of the pelvic area, not the back. A softer mattress may not provide the support needed to maintain proper alignment of the spine and may contribute to back pain.

How would the physiologic process of the sexual response best be characterized? a. Coitus, masturbation, and fantasy b. Myotonia and vasocongestion c. Erection and orgasm d. Excitement, plateau, and orgasm

ANS: B Physiologically, sexual response can be analyzed in terms of two processes: vasocongestion and myotonia. Coitus, masturbation, and fantasy are forms of stimulation for the physical manifestation of the sexual response. Erection and orgasm occur in two of the four phases of the sexual response cycle. Excitement, plateau, and orgasm are three of the four phases of the sexual response cycle.

Significant advances have been made with most reproductive technologies. Which improvement has resulted in increased success related to preimplantation genetic diagnosis? a. Embryos are transferred at the cleavage stage. b. Embryos are transferred at the blastocyst stage. c. More than two embryos can be transferred at a time. d. Two cells are removed from each embryo.

ANS: B Preimplantation genetic diagnosis can be performed on a single cell removed from each embryo after 3 to 4 days. With the availability of extended culture mediums, embryos are transferred at the blastocyst stage (day 5), which increases the chance of a live birth, compared with the older practice of transferring embryos at the cleavage stage (day 3). No more than two embryos should be transferred at a time.

Ideally, when should prenatal care begin? a. Before the first missed menstrual period b. After the first missed menstrual period c. After the second missed menstrual period d. After the third missed menstrual period

ANS: B Prenatal care should begin soon after the first missed menstrual period. This offers the greatest opportunities to ensure the health of the expectant mother and her infant. Prenatal care before missing the first menstrual period is too early. It is unlikely the woman is even aware of the pregnancy. Ideally, prenatal visits should begin soon after the first period is missed. Beginning prenatal care after the third missed menstrual period is too late. The woman will have completed the first trimester by that time.

Which statement regarding the probable signs of pregnancy is most accurate? a. Determined by ultrasound b. Observed by the health care provider c. Reported by the client d. Confirmed by diagnostic tests

ANS: B Probable signs are those detected through trained examination. Fetal visualization is a positive sign of pregnancy. Presumptive signs are those reported by the client. The term diagnostic tests is open for interpretation. To diagnose pregnancy, one would have to see positive signs of pregnancy.

Which information is most important to provide to the client interested in using the lactational amenorrhea method for contraception? a. LAM is effective until the infant is 9 months of age. b. This popular method of birth control works best if the mother is exclusively breastfeeding. c. Its typical failure rate is 5%. d. Feeding intervals should be 6 hours during the day.

ANS: B The LAM works best if the mother is exclusively or almost exclusively breastfeeding. Disruption of the breastfeeding pattern increases the risk of pregnancy. After the infant is 6 months of age or menstrual flow has resumed, effectiveness decreases. The typical failure rate is 1% to 2%. Feeding intervals should be no greater than 4 hours during the day and 6 hours at night.

Which body part both protects the pelvic structures and accommodates the growing fetus during pregnancy? a. Perineum b. Bony pelvis c. Vaginal vestibule d. Fourchette

ANS: B The bony pelvis protects and accommodates the growing fetus. The perineum covers the pelvic structures. The vaginal vestibule contains openings to the urethra and vagina. The area of thin, flat tissue called the fourchette is formed by the labia minor and is found underneath the vaginal opening.

A woman who is 6 months pregnant has sought medical attention, saying she fell down the stairs. What scenario would cause an emergency department nurse to suspect that the woman has been a victim of intimate partner violence (IPV)? a. The woman and her partner are having an argument that is loud and hostile. b. The woman has injuries on various parts of her body that are in different stages of healing. c. Examination reveals a fractured arm and fresh bruises. d. She avoids making eye contact and is hesitant to answer questions.

ANS: B The client may have multiple injuries in various stages of healing that indicates a pattern of violence. An argument is not always an indication of battering. A fractured arm and fresh bruises could be caused by the reported fall and do not necessarily indicate IPV. It may be normal for the woman to be reticent and have a dull affect.

The unique muscle fibers that constitute the uterine myometrium make it ideally suited for what? a. Menstruation b. Birth process c. Ovulation d. Fertilization

ANS: B The myometrium is made up of layers of smooth muscle that extend in three directions. These muscles assist in the birth process by expelling the fetus, ligating blood vessels after birth, and controlling the opening of the cervical os. These muscle fibers have no contribution to the process of menstruation, ovulation or fertilization.

A 62-year-old woman has not been to the clinic for an annual examination for 5 years. The recent death of her husband reminded her that she should come for a visit. Her family physician has retired, and she is going to see the women's health nurse practitioner for her visit. What should the nurse do to facilitate a positive health care experience for this client? a. Remind the woman that she is long overdue for her examination and that she should come in annually. b. Carefully listen and allow extra time for this woman's health history interview. c. Reassure the woman that a nurse practitioner is just as good as her old physician. d. Encourage the woman to talk about the death of her husband and her fears about her own death.

ANS: B The nurse has an opportunity to use reflection and empathy while listening, as well as ensure an open and caring communication. Scheduling a longer appointment time may be necessary because older women may have longer histories or may need to talk. A respectful and reassuring approach to caring for women older than age 50 years can help ensure that they continue to seek health care. Reminding the woman about her overdue examination, reassuring the woman that she has a good practitioner, and encouraging conversation about the death of her husband and her own death are not the best approaches.

During the physical examination of a client beginning prenatal care, which initial action is most important for the nurse to perform? a. Only women who show physical signs or meet the sociologic profile should be assessed for physical abuse. b. The client should empty her bladder before the pelvic examination. c. The distribution, amount, and quality of body hair are of no particular importance. d. The size of the uterus is discounted in the initial examination because it will be increasing in size during the second trimester.

ANS: B The nurse should instruct the client to empty her bladder. An empty bladder facilitates the examination and also provides an opportunity to obtain a urine sample for a number of tests. All women should be assessed for a history of physical abuse, particularly because the likelihood of abuse increases during pregnancy. Noting body hair is important because body hair reflects nutritional status, endocrine function, and hygiene. Particular attention is paid to the size of the uterus because it is an indication of the duration of gestation.

Which condition is likely the biggest risk for the pregnant client? a. Preeclampsia b. Intimate partner violence (IPV) c. Diabetes d. Abnormal Pap test

ANS: B The prevalence of IPV during pregnancy is estimated at 6% of all pregnant women. The risk for IPV and even IPV-related homicide is more common than all of the other pregnancy-related conditions. Although preeclampsia poses a risk to the health of the pregnant client, it is less common than IPV. Gestational diabetes continues to be a complication of pregnancy; however, it is less common than IPV during pregnancy. Some women are at risk for an abnormal Pap screening during pregnancy, but this finding is not as common as IPV.

Cardiac output increases up to 50% by the 32nd week of pregnancy. What is the rationale for this change? a. To compensate for the decreased renal plasma flow b. To provide adequate perfusion of the placenta c. To eliminate metabolic wastes of the mother d. To prevent maternal and fetal dehydration

ANS: B The primary function of increased vascular volume is to transport oxygen and nutrients to the fetus via the placenta. Renal plasma flow increases during pregnancy. Assisting with pulling metabolic wastes from the fetus for maternal excretion is one purpose of the increased vascular volume.

Of which physiologic alteration of the uterus during pregnancy is it important for the nurse to alert the patient? a. Lightening occurs near the end of the second trimester as the uterus rises into a different position. b. Woman's increased urinary frequency in the first trimester is the result of exaggerated uterine anti-reflexion caused by softening. c. Braxton Hicks contractions become more painful in the third trimester, particularly if the woman tries to exercise. d. Uterine souffle is the movement of the fetus.

ANS: B The softening of the lower uterine segment is called the Hegar sign. In this position, the uterine fundus presses on the bladder, causing urinary frequency that is a normal change of pregnancy. Lightening occurs in the last 2 weeks of pregnancy, when the fetus descends. Braxton Hicks contractions become more defined in the final trimester but are not painful. Walking or exercise usually causes them to stop. The uterine souffle is the sound made by blood in the uterine arteries; it can be heard with a fetal stethoscope.

Importantly, the nurse must be aware of which information related to the use of intrauterine devices (IUDs)? a. Return to fertility can take several weeks after the device is removed. b. Copper IUDs can serve as an emergency contraception under certain situations. c. IUDs offer the same protection against STIs as the diaphragm. d. Consent forms are not needed for IUD insertion.

ANS: B The woman has up to 5 days to insert the IUD after unprotected sex. The return to fertility is immediate after the removal of the IUD. IUDs offer no protection against STIs. A consent form is required for insertion, as is a negative pregnancy test.

Which statement by the client indicates that she understands breast self-examination (BSE)? a. "I will examine both breasts in two different positions." b. "I will examine my breasts 1 week after my menstrual period starts." c. "I will examine only the outer upper area of the breast." d. "I will use the palm of the hand to perform the examination."

ANS: B The woman should examine her breasts when hormonal influences are at their lowest level. The client should be instructed to use four positions: standing with arms at her sides, standing with arms raised above her head, standing with hands pressed against hips, and lying down. The entire breast needs to be examined, including the outer upper area. The client should use the sensitive pads of the middle three fingers.

Which treatment regime would be most appropriate for a client who has been recently diagnosed with acute pelvic inflammatory disease (PID)? a. Oral antiviral therapy b. Bed rest in a semi-Fowler position c. Antibiotic regimen continued until symptoms subside d. Frequent pelvic examination to monitor the healing progress

ANS: B The woman with acute PID should be on bed rest in a semi-Fowler position. Broad-spectrum antibiotics are used; antiviral therapy is ineffective. Antibiotics must be taken as prescribed, even if symptoms subside. Few pelvic examinations should be conducted during the acute phase of the disease.

The nurse is providing health education to a pregnant client regarding the cardiovascular system. Which information is correct and important to share? a. A pregnant woman experiencing disturbed cardiac rhythm requires close medical and obstetric observation no matter how healthy she may appear otherwise. b. Changes in heart size and position and increases in blood volume create auditory changes from 20 weeks of gestation to term. c. Palpitations are twice as likely to occur in twin gestations. d. All of the above changes will likely occur.

ANS: B These auscultatory changes should be discernible after 20 weeks of gestation. A healthy woman with no underlying heart disease does not need any therapy. The maternal heart rate increases in the third trimester, but palpitations may not necessarily occur, let alone double. Auditory changes are discernible at 20 weeks of gestation.

Which statement regarding multifetal pregnancy is incorrect? a. The expectant mother often develops anemia because the fetuses have a greater demand for iron. b. Twin pregnancies come to term with the same frequency as single pregnancies. c. The mother should be counseled to increase her nutritional intake and gain more weight. d. Backache and varicose veins often are more pronounced with a multifetal pregnancy.

ANS: B Twin pregnancies often end in prematurity. Serious efforts should be made to bring the pregnancy to term. A woman with a multifetal pregnancy often develops anemia, suffers more or worse backache, and needs to gain more weight. Counseling is needed to help her adjust to these conditions.

Which test is performed around the time of ovulation to diagnose the basis of infertility? a. Hysterosalpingogram b. Vaginal ultrasonography c. Laparoscopy d. Follicle-stimulating hormone (FSH) level

ANS: B Ultrasonography is performed around the time of ovulation to assess pelvic structures for abnormalities, to verify follicular development, and to assess the thickness of the endometrium. A hysterosalpingogram is scheduled 2 to 5 days after menstruation to avoid flushing a potentially fertilized ovum out through a uterine tube into the peritoneal cavity. Laparoscopy is usually scheduled early in the menstrual cycle. Hormone analysis is performed to assess endocrine function of the hypothalamic-pituitary-ovarian axis when menstrual cycles are absent or irregular.

Which statement best describes the rationale for the physiologic anemia that occurs during pregnancy? a. Physiologic anemia involves an inadequate intake of iron. b. Dilution of hemoglobin concentration occurs in pregnancy with physiologic anemia. c. Fetus establishes the iron stores. d. Decreased production of erythrocytes occur.

ANS: B When blood volume expansion is more pronounced and occurs earlier than the increase in red blood cells, the woman has physiologic anemia, which is the result of the dilution of hemoglobin concentration rather than inadequate hemoglobin. An inadequate intake of iron may lead to true anemia. The production of erythrocytes increases during pregnancy.

Which statement made by a lactating woman leads the nurse to believe that the client might have lactose intolerance? a. "I always have heartburn after I drink milk." b. "If I drink more than a cup of milk, I usually have abdominal cramps and bloating." c. "Drinking milk usually makes me break out in hives." d. "Sometimes I notice that I have bad breath after I drink a cup of milk."

ANS: B Lactose intolerance, which is an inability to digest milk sugar because of a lack of the enzyme lactose in the small intestine, is a problem that interferes with milk consumption. Milk consumption may cause abdominal cramping, bloating, and diarrhea in such people, although many lactose-intolerant individuals can tolerate small amounts of milk without symptoms. A woman with lactose intolerance is more likely to experience bloating and cramping, not heartburn. A client who breaks out in hives after consuming milk is more likely to have a milk allergy and should be advised to simply brush her teeth after consuming dairy products.

A 31-year-old woman believes that she may be pregnant. She took an over-the-counter (OTC) pregnancy test 1 week ago after missing her period; the test was positive. During her assessment interview, the nurse asks about the woman's last menstrual period and whether she is taking any medications. The client states that she takes medicine for epilepsy. She has been under considerable stress lately at work and has not been sleeping well. Her physical examination does not indicate that she is pregnant. She has an ultrasound scan, which confirms that she is not pregnant. What is the most likely cause of the false-positive pregnancy test result? a. The pregnancy test was taken too early. b. Anticonvulsant medications may cause the false-positive test result. c. The woman has a fibroid tumor. d. She has been under considerable stress and has a hormone imbalance.

ANS: B Anticonvulsants may cause false-positive pregnancy test results. OTC pregnancy tests use enzyme-linked immunosorbent assay (ELISA) technology, which can yield positive results as soon as 4 days after implantation. Implantation occurs 6 to 10 days after conception. If the woman were pregnant, then she would be into her third week at this point (having missed her period 1 week ago0. Fibroid tumors do not produce hormones and have no bearing on human chorionic gonadotropin (hCG) pregnancy tests. Although stress may interrupt normal hormone cycles (menstrual cycles), it does not affect hCG levels or produce positive pregnancy test results.

The client is instructed to place her thumb and forefinger on the areola and gently press inward. What is the purpose of this exercise? a. To check the sensitivity of the nipples b. To determine whether the nipple is everted or inverted c. To calculate the adipose buildup in the abdomen d. To see whether the fetus has become inactive

ANS: B Sometimes known as the pinch test, this exercise is used to determine whether the nipple is everted or inverted. Nipples must be everted to allow breastfeeding to occur naturally. The pinch does not determine the level of sensitivity of the nipples, nor is it not used to determine the level of adipose tissue in the abdomen. Fetal activity is not determined by using the pinch test.

Which finding in the urinalysis of a pregnant woman is considered a variation of normal? a. Proteinuria b. Glycosuria c. Bacteria in the urine d. Ketones in the urine

ANS: B Small amounts of glucose may indicate physiologic spilling. The presence of protein could indicate kidney disease or preeclampsia. Urinary tract infections are associated with bacteria in the urine. An increase in ketones indicates that the patient is exercising too strenuously or has an inadequate fluid and food intake.

Which clinical presentation should the nurse assess for if a pregnant client is experiencing a Jarisch-Herxheimer reaction? (Select all that apply) a. Vomiting and diarrhea b. Headache c. Preterm labor contractions d. CHOICE BLANK e. Bright red vaginal bleeding Arthralgia

ANS: B C, E Clients treated for syphilis with penicillin may experience a Jarisch-Herxheimer reaction. The reaction is an acute febrile reaction that occurs within the first 24 hours of treatment and is accompanied by headache, myalgias, and arthralgia. If the client is pregnant, then she is at risk for preterm labor and birth. Neither vaginal bleeding, vomiting nor diarrhea are anticipated.

Which sexual behaviors are associated with high exposure to a sexually transmitted infection (STI)? (Select all that apply.) a. Monogamy b. Unprotected anal intercourse c. Multiple sex partners d. Dry kissing e. Self-masturbation

ANS: B, C Engaging in unprotected anal intercourse or having multiple sex partners increases the exposure risk and the possibility of acquiring an STI. Dry kissing self-masturbation, and monogamy are considered safe sexual practices.

Women who have undergone an oophorectomy, have ovarian failure, or a genetic defect may be eligible to receive donor oocytes (eggs). Which statements regarding oocyte donation are accurate? (Select all that apply.) a. Donor is inseminated with semen from the parent. b. Donor eggs can be fertilized with the male partner's sperm. c. Donors are under 35 years of age. d. Recipient undergoes hormonal stimulation. e. Ovum is placed into a surrogate.

ANS: B, C, D Oocyte donation is usually provided by healthy women under the age of 35 years, who are recruited and paid to undergo ovarian stimulation and oocyte retrieval. The donor eggs are fertilized in a laboratory with the male partner's sperm. The woman undergoes hormonal stimulation to allow the development of the uterine lining. Embryos are then transferred. A donor that is inseminated with the male partner's semen or receives the fertilized ovum and then carries it to gestation is known as a surrogate mother.

Women of all ages will receive substantial and immediate benefits from smoking cessation. The process is not easy, and most people have attempted to quit numerous times before achieving success. Which organizations provide self-help and smoking cessation materials? (Select all that apply.) a. Leukemia and Lymphoma Society b. March of Dimes c. American Cancer Society d. American Lung Association e. Easter Seals

ANS: B, C, D The March of Dimes, the American Lung Association, and the American Cancer Society have self-help materials available. The Leukemia and Lymphoma Society support research for these two types of cancer. Easter Seals is best known for its work with disabled children.

What are the responsibilities of the nurse who suspects or confirms any type of violence against a woman? (Select all that apply.) a. Report the incident to legal authorities. b. Provide resources for domestic violence shelters. c. Call a client advocate who can assist in the client's decision about what actions to take. d. Accurately and concisely document the incident (or findings) in the client's record. e. Reassure and support the client.

ANS: B, C, D, E Domestic violence is considered a crime in all states; however, mandatory reporting remains controversial. Nurses must become knowledgeable on the laws that apply in the state in which they practice. Caring for a client who may be a victim of domestic abuse is an ideal opportunity to provide the woman with information for safe houses or support groups for herself and her children. The nurse may assist in reaching out to a client advocate, which often occurs when potential legal action is taken or if the woman is seeking shelter. Documentation must be accurate and timely to be useful to the client later in court if she chooses to press charges. The primary functions for the nurse are to reassure the client and to provide her with emotional support.

What are some common characteristics of a potential male batterer? (Select all that apply.) a. High level of assertiveness b. Low frustration tolerance c. Substance abuse problems d. High level of empathy e. Personality disorders

ANS: B, C, E Substance abuse and personality disorders are often observed in batterers. Typically, the batterer has deficits in assertiveness towards others with exception of the battered female. Batterers usually have a low frustration level (i.e., they easily lose their temper). Batterers characteristically demonstrate an inability to demonstrate empathy (the ability to understand and share the feelings of another).

The number of routine laboratory tests during follow-up visits is limited; however, those that are performed are essential. Which statements regarding group B Streptococcus (GBS) testing are correct? (Select all that apply.) a. Performed between 32 and 34 weeks of gestation. b. Performed between 35 and 37 weeks of gestation. c. All women should be tested. d. Only women planning a vaginal birth should be tested. e. Women with a history of GBS should be retested.

ANS: B, C, E GBS testing is recommended between 35 and 37 weeks of gestation; cultures collected earlier will not accurately predict the presence of GBS at birth. All women should be tested, even those planning an elective cesarean birth. Membranes may rupture early, requiring prophylactic antibiotics. Clients with a history of GBS should be retested.

Which statements regarding the Human papillomavirus (HPV) are accurate? (Select all that apply.) HPV infections: a. HPV is thought to be less common in pregnant women than in women who are not pregnant. b. HPV is thought to be more common in pregnant than in non-pregnant women. c. HVP was previously referred to as genital warts. d. HVP was previously referred to as herpes. e. HVP is thought to sometimes cause cervical cancer.

ANS: B, C, E HPV infections are thought to be more common in pregnant women than in women who are not pregnant, with an increase in incidence from the first trimester to the third trimester. HPV, formerly called venereal or genital warts, is an STI with more than 30 known serotypes, several of which are associated with cervical cancer.

While educating the client regarding the risks and benefits of a vasectomy, which information should the nurse include? (Select all that apply.) a. Sterilization should be performed under general anesthesia. b. Pain, bleeding, and infection are possible complications. c. For several months, pregnancy may still be possible. d. Vasectomy may affect potency. e. Secondary sex characteristics are unaffected.

ANS: B, C, E Vasectomy is the most commonly used procedure for male sterilization and is performed on an outpatient basis under local anesthesia. Pain, bleeding, swelling, and infection are considered complications. Reversal is generally unsuccessful; however, it may take several weeks to months for all sperm to be cleared from the sperm ducts. Another form of contraception is necessary until the sperm counts are zero. Vasectomy has no effect on potency, and secondary sex characteristics are not affected.

What are the two primary functions of the ovary? (Select all that apply.) a. Normal female development b. Ovulation c. Sexual response d. Hormone production e. Sex hormone release

ANS: B, D The two functions of the ovaries are ovulation and hormone production. The presence of ovaries does not guarantee normal female development. The ovaries produce estrogen, progesterone, and androgen. Ovulation is the release of a mature ovum from the ovary. Sexual response is a feedback mechanism involving the hypothalamus, anterior pituitary gland, and ovaries.

The nurse guides a woman to the examination room and asks her to remove her clothes and put on an examination gown with the front open. The woman replies, "I have special undergarments that I do not remove for religious reasons." Which is the most appropriate response from the nurse? a. "You can't have an examination without removing all your clothes." b. "I'll ask the physician to modify the examination." c. "I'll explain the examination procedure, and then we can discuss how you can comfortably have your examination." d. "I have no idea how we can accommodate your beliefs."

ANS: C Explaining the examination procedure reflects cultural competence by the nurse and shows respect for the woman's religious practices. The nurse must respect the rich and unique qualities that cultural diversity brings to individuals. The examination can be modified to ensure that modesty is maintained. In recognizing the value of cultural differences, the nurse N R I G B.C M USNT O can modify the plan of care to meet the needs of each woman. Telling the client that her religious practices are different or strange is inappropriate and disrespectful to the client.

What should the nurse be cognizant of concerning the client's reordering of personal relationships during pregnancy? a. Because of the special motherhood bond, a woman's relationship with her mother is even more important than with her partner. b. Nurses need not get involved in any sexual issues the couple has during pregnancy, particularly if they have trouble communicating them to each other. c. Women usually express two major relationship needs during pregnancy: feeling loved and valued and having the child accepted by the partner. d. The woman's sexual desire is likely to be highest in the first trimester because of the excitement and because intercourse is physically easier.

ANS: C Love and support help a woman feel better about her pregnancy. The most important person to the pregnant woman is usually her partner. Nurses can facilitate communication between partners about sexual matters if, as is common, they are nervous about expressing their worries and feelings to one another. The second trimester is the time when a woman's sense of well-being, along with certain physical changes, increases her desire for sex. Sexual desire is down in the first and third trimesters.

Preconception and prenatal care have become important components of women's health. What is the guiding principal of preconception care? a. Ensure that pregnancy complications do not occur. b. Identify the woman who should not become pregnant. c. Encourage healthy lifestyles for families desiring pregnancy. d. Ensure that women know about prenatal care.

ANS: C Preconception counseling guides couples in how to avoid unintended pregnancies, how to identify and manage risk factors in their lives and in their environment, and how to identify healthy behaviors that promote the well-being of the woman and her potential fetus. Preconception care does not ensure that pregnancy complications will not occur. In many cases, problems can be identified and treated and may not recur in subsequent pregnancies. For many women, counseling can allow behavior modification before any damage is done, or a woman can make an informed decision about her willingness to accept potential hazards. If a woman is seeking preconception care, then she is likely aware of prenatal care.

In the 1970s, the rape-trauma syndrome (RTS) was identified as a cluster of symptoms and related behaviors observed in the weeks and months after an episode of rape. Researchers identified three phases related to this condition. Which phase is not displayed in a client with RTS? a. Acute Phase: Disorganization b. Outward Adjustment Phase c. Shock/Disbelief: Disorientation Phase d. Long-Term Process: Reorganization Phase

ANS: C Shock, disbelief, or disorientation is a component of the Acute Phase. The rape survivor feels embarrassed, degraded, fearful, and angry. She may feel unclean and want to bathe and douche repeatedly, even though doing so may destroy evidence. The victim relives the scene over and over in her mind, thinking of things she "should have done." During the Outward Adjustment Phase, the victim may appear to have resolved her crisis and return to activities of daily living and work. Other women may move, leave their job, and buy a weapon to protect themselves. Disorientation is a reaction during which the victim may feel disoriented, have difficulty concentrating, or have poor recall. The Long-Term Process is the reorganization phase. This recovery phase may take years and may be difficult and painful.

While obtaining a diet history, the nurse might be told that the expectant mother has cravings for ice chips, cornstarch, and baking soda. Which nutritional problem does this behavior indicate? a. Preeclampsia b. Pyrosis c. Pica d. Purging

ANS: C The consumption of foods low in nutritional value or of nonfood substances (e.g., dirt, laundry starch) is called pica. Preeclampsia is a vasospastic disease process encountered after 20 weeks of gestation. Characteristics of preeclampsia include increasing hypertension, proteinuria, and hemoconcentration. Pyrosis is a burning sensation in the epigastric region, otherwise known as heartburn. Purging refers to self-induced vomiting after consuming large quantities of food.

Which phase of the endometrial cycle best describes a heavy, velvety soft, fully matured endometrium? a. Menstrual b. Proliferative c. Secretory d. Ischemic

ANS: C The secretory phase extends from the day of ovulation to approximately 3 days before the next menstrual cycle. During this secretory phase, the endometrium becomes fully mature again. During the menstrual phase, the endometrium is shed. The proliferative phase is a period of rapid growth. During the ischemic phase, the blood supply is blocked, and necrosis develops.

A pregnant woman's diet consists almost entirely of whole grain breads and cereals, fruits, and vegetables. Which dietary requirement is the nurse most concerned about? a. Calcium b. Protein c. Vitamin B12 d. Folic acid

ANS: C A pregnant woman's diet is consistent with that followed by a strict vegetarian (vegan). Vegans consume only plant products. Because vitamin B12 is found in foods of animal origin, this diet is deficient in vitamin B12. Depending on the woman's food choices, a pregnant woman's diet may be adequate in calcium. Protein needs can be sufficiently met by a vegetarian diet. The nurse should be more concerned with the woman's intake of vitamin B12 attributable to her dietary restrictions. Folic acid needs can be met by enriched bread products.

Which statement regarding the development of the respiratory system is a high priority for the nurse to understand? a. The respiratory system does not begin developing until after the embryonic stage. b. The infant's lungs are considered mature when the L/S ratio is 1:1, at approximately 32 weeks of gestation. c. Maternal hypertension can reduce maternal-placental blood flow, accelerating lung maturity. d. Fetal respiratory movements are not visible on ultrasound scans until at least 16 weeks of gestation.

ANS: C A reduction in placental blood flow stresses the fetus, increases blood levels of corticosteroids, and thus accelerates lung maturity. The development of the respiratory system begins during the embryonic phase and continues into childhood. The infant's lungs are considered mature when the L/S ratio is 2:1, at approximately 35 weeks of gestation. Lung movements have been visualized on ultrasound scans at 11 weeks of gestation.

Which questionnaire would be best for the nurse to use when screening an adolescent client for an eating disorder? a. Four Cs b. Dietary Guidelines for Americans c. SCOFF screening tool d. Dual-energy x-ray absorptiometry (DEXA) scan

ANS: C A screening tool specifically developed to identify eating disorders uses the acronym SCOFF. Each question scores 1 point. A score of 2 or more indicates that the client may have anorexia nervosa or bulimia. The letters represent the following questions: Do you make yourself Sick because you feel too full? Do you worry about loss of Control over the amount that you eat? Have you recently lost more than One stone (14 pounds) in a 3-month period? Do you think that you are too Fat, even if others think you are thin? Does Food dominate your life? The 4 Cs are used to determine cultural competence. Dietary Guidelines for Americans provide nutritional guidance for all, not only for those with eating disorders. The DEXA scan is used to determine bone density.

Pregnant adolescents are at greater risk for decreased body mass index (BMI) and "fad" dieting with which condition? a. Obesity b. Gestational diabetes c. Low-birth-weight babies d. High-birth-weight babies

ANS: C Adolescents tend to have lower BMIs. In addition, the fetus and the still-growing mother appear to compete for nutrients. These factors, along with inadequate weight gain, lend themselves to a higher incidence of low-birth-weight babies. Obesity is associated with a higher-than-normal BMI. Unless the teenager has type 1 diabetes, an adolescent with a low BMI is less likely to develop gestational diabetes. High-birth-weight or large-for-gestational age (LGA) babies are most often associated with gestational diabetes.

Which client would be an ideal candidate for injectable progestins such as medroxyprogesterone acetate as a contraceptive choice? a. The ideal candidate wants menstrual regularity and predictability. b. The client has a history of thrombotic problems or breast cancer. c. The ideal candidate has difficulty remembering to take oral contraceptives daily. d. The client is homeless or mobile and rarely receives health care.

ANS: C Advantages of medroxyprogesterone acetate includes its contraceptive effectiveness, compared with the effectiveness of combined oral contraceptives, and the requirement of only four injections a year. The disadvantages of injectable progestins are prolonged amenorrhea and uterine bleeding. The use of injectable progestin carries an increased risk of venous thrombosis and thromboembolism. To be effective, injections must be administered every 11 to 13 weeks. Access to health care is necessary to prevent pregnancy or potential complications.

Some pregnant clients may report changes in their voice and impaired hearing. What should the nurse explain to the client concerning these findings? a. Voice changes are caused by decreased estrogen levels. b. Displacement of the diaphragm results in thoracic breathing. c. These changes are the results of congestion and swelling of the upper respiratory tract. d. Increased blood volume causes changes in the voice.

ANS: C Although the diaphragm is displaced, and the volume of blood is increased, neither causes changes in the voice nor impairs hearing. The key is that estrogen levels increase, not decrease, which causes the upper respiratory tract to become more vascular, which produces swelling and congestion in the nose and ears and therefore voice changes and impaired hearing.

A woman tells the nurse that she thinks she has a vaginal infection, and has been using an over-the-counter cream for the past 2 days to treat it. How should the nurse initially respond? a. Determine when she first noticed the symptoms. b. Reassure the woman that using vaginal cream is not a problem for the examination. c. Ask the woman to describe the symptoms that indicate to her that she has a vaginal infection. d. Ask the woman to reschedule the appointment for the examination.

ANS: C An important element of the health history and physical examination is the client's description of any symptoms she may be experiencing. The best response is for the nurse to inquire about the symptoms the woman is experiencing. While relevant, when the symptoms began is not as important as what the symptoms are. Women should not douche, use vaginal medications, or have sexual intercourse for 24 to 48 hours before obtaining a Pap test. Although the woman may need to reschedule a visit for her Pap test, her current symptoms should still be addressed.

A pregnant woman at 25 weeks of gestation tells the nurse that she dropped a pan last week and her baby jumped at the noise. Which response by the nurse is most accurate? a. "That must have been a coincidence; babies can't respond like that." b. "The fetus is demonstrating the aural reflex." c. "Babies respond to sound starting at approximately 24 weeks of gestation." d. "Let me know if it happens again; we need to report that to your midwife."

ANS: C Babies respond to external sound starting at approximately 24 weeks of gestation. Acoustic stimulations can evoke a fetal heart rate response. There is no such thing as an aural reflex. The last statement is inappropriate and may cause undue psychologic alarm to the client.

What is the drug of choice for the treatment of gonorrhea? a. Penicillin G b. Tetracycline c. Ceftriaxone d. Acyclovir

ANS: C Ceftriaxone is effective for the treatment of all gonococcal infections. Penicillin is used to treat syphilis. Tetracycline is prescribed for chlamydial infections. Acyclovir is used to treat herpes genitalis.

Which sexually transmitted infection (STI) is the most commonly reported in American women? a. Gonorrhea b. Syphilis c. Chlamydia d. Candidiasis

ANS: C Chlamydia is the most common STI in women in the United States and one of the most common causes of pelvic inflammatory disease (PID). Gonorrhea is probably the oldest communicable disease in the United States and second to Chlamydia in reported conditions. Syphilis has reemerged as a common STI, affecting black women more than any other ethnic or racial group. Candidiasis is a relatively common fungal infection.

Nurses who provide care to victims of intimate partner violence (IPV) should be keenly aware of what? a. Relationship violence usually consists of a single episode that the couple can put behind them. b. Violence often declines or ends with pregnancy. c. Financial coercion is considered part of IPV. d. Battered women are generally poorly educated and come from a deprived social background.

ANS: C Economic coercion may accompany physical assault and psychologic attacks. IPV almost always follows an escalating pattern. It rarely ends with a single episode of violence. IPV often begins with and escalates during pregnancy. It may include both psychologic attacks and economic coercion. Race, religion, social background, age, and education level are not significant factors in differentiating women at risk.

Which sexual transmitted infection (STI) does not respond well to antibiotic therapy? a. Chlamydia b. Gonorrhea c. Genital herpes d. Syphilis

ANS: C Genital herpes is a chronic and recurring viral infection for which no known cure is available; therefore, it does not respond to antibiotics. Chlamydia is a bacterial infection that is treated with doxycycline or azithromycin. Gonorrhea is a bacterial infection that is treated with any of several antibiotics. Syphilis is a bacterial infection that is treated with penicillin.

Which condition is the most life-threatening virus to the fetus and neonate? a. Hepatitis A virus (HAV) b. Herpes simplex virus (HSV) c. Hepatitis B virus (HBV) d. Cytomegalovirus (CMV)

ANS: C HBV is the most life-threatening viral condition to the fetus and neonate. HAV is not the most threatening to the fetus nor is HSV the most threatening to the neonate. Although serious, CMV is not the most life-threatening viral condition to the fetus.

With regard to weight gain during pregnancy, the nurse should be aware of which important information? a. In pregnancy, the woman's height is not a factor in determining her target weight. b. Obese women may have their health concerns, but their risk of giving birth to a child with major congenital defects is the same as with women of normal weight. c. Women with inadequate weight gain have an increased risk of delivering a preterm infant with intrauterine growth restriction (IUGR). d. Greater than expected weight gain during pregnancy is almost always attributable to old-fashioned overeating.

ANS: C IUGR is associated with women with inadequate weight gain. The primary factor in making a weight gain recommendation is the appropriateness of the prepregnancy weight for the woman's height. Obese women are twice as likely as women of normal weight to give birth to a child with major congenital defects. Overeating is only one of several likely causes.

In the acronym BRAIDED, which letter is used to identify the key components of informed consent that the nurse must document? a. B stands for birth control. b. R stands for reproduction. c. A stands for alternatives. d. I stands for ineffective.

ANS: C In the acronym BRAIDED, A stands for alternatives and information about other viable methods. B stands for benefits and information about the advantages of a particular birth control method and its success rates. R stands for risks and information about the disadvantages of a particular method and its failure rates. I stands for inquiries and the opportunity to ask questions.

Which minerals and vitamins are usually recommended as a supplement in a pregnant client's diet? a. Fat-soluble vitamins A and D b. Water-soluble vitamins C and B6 c. Iron and folate d. Calcium and zinc

ANS: C Iron should generally be supplemented, and folic acid supplements are often needed because folate is so important in pregnancy. Fat-soluble vitamins should be supplemented as a medical prescription, as vitamin D might be for lactose-intolerant women. Water-soluble vitamin C is sometimes naturally consumed in excess; vitamin B6 is prescribed only if the woman has a very poor diet; and zinc is sometimes supplemented. Most women get enough calcium.

The nurse is providing education to a client regarding the normal changes of the breasts during pregnancy. Which statement regarding these changes is correct? a. The visibility of blood vessels that form an intertwining blue network indicates a possible infection of the tubercles. b. The mammary glands do not develop until 2 weeks before labor. c. Lactation is inhibited until the progesterone level declines after birth. d. Colostrum is the yellowish oily substance used to lubricate the nipples for breastfeeding.

ANS: C Lactation is inhibited until after birth due to progesterone levels. The visible blue network of blood vessels is a normal outgrowth of a richer blood supply. The mammary glands are functionally complete by mid-pregnancy. Colostrum is a creamy white-to-yellow pre-milk fluid that can be expressed from the nipples before birth.

The nurse is providing contraceptive instruction to a young couple who are eager to learn. The nurse should be cognizant of which information regarding the natural family planning method? a. The natural family planning method is the same as coitus interruptus or "pulling out." b. This contraception method uses the calendar method to align the woman's cycle with the natural phases of the moon. c. This practice is the only contraceptive method acceptable to the Roman Catholic Church. d. The natural family planning method relies on barrier methods during the fertility phases.

ANS: C Natural family planning is the only contraceptive practice acceptable to the Roman Catholic Church. "Pulling out" is not the same as periodic abstinence, another name for natural family planning. The phases of the moon are not part of the calendar method or any method. Natural family planning is another name for periodic abstinence, which is the accepted way to pass safely through the fertility phases without relying on chemical or physical barriers.

What is the primary reason why a woman who is older than 35 years may have difficulty achieving pregnancy? a.Personal risk behaviors influence fertility. b. Mature women have often used contraceptives for an extended time. c. Her ovaries may be affected by the aging process. d. Pre-pregnancy medical attention is lacking.

ANS: C Once the mature woman decides to conceive, a delay in becoming pregnant may occur because of the normal aging of the ovaries. Older adults participate in fewer risk behaviors than younger adults. The past use of contraceptives is not the problem. Pre-pregnancy medical care is both available and encouraged and has limited effect on initiating pregnancy.

Which clinical finding in a primiparous client at 32 weeks of gestation might be an indication of anemia? a. Ptyalism b. Pyrosis c. Pica d. Decreased peristalsis

ANS: C Pica (a desire to eat nonfood substances) is an indication of iron deficiency and should be evaluated. Cravings include ice, clay, and laundry starch. Ptyalism (excessive salivation), pyrosis (heartburn), and decreased peristalsis are normal findings.

Once the Human immunodeficiency virus (HIV) has entered the body, what is the time frame for seroconversion to HIV positivity? a. 6 to 10 days b. 2 to 4 weeks c. 6 to 12 weeks d. 6 months

ANS: C Seroconversion to HIV positivity usually occurs within 6 to 12 weeks after the virus has entered the body. Both 6 to 10 days and 2 to 4 weeks are too short for seroconversion to HIV positivity to occur, and 6 months is too long.

The measurement of lecithin in relation to sphingomyelin (lecithin/sphingomyelin [L/S] ratio) is used to determine fetal lung maturity. Which ratio reflects fetal maturity of the lungs? a. 1.4:1 b. 1.8:1 c. 2:1 d. 1:1

ANS: C The L/S ratio indicates a 2:1 ratio of lecithin to sphingomyelin, which is an indicator of fetal lung maturity and occurs at approximately the middle of the third trimester. L/S ratios of 1.4:1, 1.8:1, and 1:1 each indicate immaturity of the fetal lungs.

The microscopic examination of scrapings from the cervix, endocervix, or other mucous membranes to detect premalignant or malignant cells is called what? a. Bimanual palpation b. Rectovaginal palpation c. Papanicolaou (Pap) test d. Four As procedure

ANS: C The Pap test is a microscopic examination for cancer that should be regularly performed, depending on the client's age. Bimanual palpation is a physical examination of the vagina. Rectovaginal palpation is a physical examination performed through the rectum. The four As procedure is an intervention to help a client stop smoking.

A woman's cousin gave birth to an infant with a congenital heart anomaly. The woman asks the nurse when such anomalies occur during development. Which response by the nurse is most accurate? a. "We don't really know when such defects occur." b. "It depends on what caused the defect." c. "Defects occur between the third and fifth weeks of development." d. "They usually occur in the first 2 weeks of development."

ANS: C The cardiovascular system is the first organ system to function in the developing human. Blood vessel and blood formation begins in the third week, and the heart is developmentally complete in the fifth week. "We don't really know when such defects occur" is an inaccurate statement. Regardless of the cause, the heart is vulnerable during its period of development—in the third to fifth weeks; therefore, the statement, "They usually occur in the first 2 weeks of development" is inaccurate.

What is the primary theme of the feminist perspective regarding violence against women? a. Role of testosterone as the underlying cause of men's violent behavior b. Basic human instinctual drive toward aggression c. Male dominance and coercive control over women d. Cultural norm of violence in Western society

ANS: C The contemporary social view of violence is derived from the feminist theory. With the primary theme of male dominance and coercive control, this view enhances an understanding of all forms of violence against women, including wife battering, stranger and acquaintance rape, incest, and sexual harassment in the workplace. The role of testosterone as an underlying cause of men's violent behavior, the basic human instinctual drive toward aggression, and the cultural norm of violence in Western society are not associated with the feminist perspective regarding violence against women.

Which statement regarding female sexual response is inaccurate? a. Women and men are more alike than different in their physiologic response to sexual arousal and orgasm. b. Vasocongestion is the congestion of blood vessels. c. Orgasmic phase is the final state of the sexual response cycle. d. Facial grimaces and spasms of the hands and feet are often part of arousal.

ANS: C The final state of the sexual response cycle is the resolution phase after orgasm. Men and women are surprisingly alike. Vasocongestion causes vaginal lubrication and engorgement of the genitals. Arousal is characterized by increased muscular tension (myotonia).

A client arrives for her initial prenatal examination. This is her first child. She asks the nurse, "How does my baby get air inside my uterus?" What is the correct response by the nurse? a. "The baby's lungs work in utero to exchange oxygen and carbon dioxide." b. "The baby absorbs oxygen from your blood system." c. "The placenta provides oxygen to the baby and excretes carbon dioxide into your bloodstream." d. "The placenta delivers oxygen-rich blood through the umbilical artery to the baby's abdomen."

ANS: C The placenta delivers oxygen-rich blood through the umbilical vein, not the artery, to the fetus and excretes carbon dioxide into the maternal bloodstream. The fetal lungs do not function as respiratory gas exchange in utero. The baby does not simply absorb oxygen from a woman's blood system; rather, blood and gas transport occur through the placenta.

Dental care during pregnancy is an important component of good prenatal care. Which instruction regarding dental health should the nurse provide? a. Regular brushing and flossing may not be necessary during early pregnancy because it may stimulate the woman who is already nauseated to vomit. A cleaning is all that is necessary. b. Dental surgery is contraindicated during pregnancy and should be delayed until after delivery. c. If dental treatment is necessary, then the woman will be most comfortable with it in the second trimester. d. If a woman has dental anxiety, then dental care may interfere with the expectant mother's need to practice conscious relaxation and to prepare for labor.

ANS: C The second trimester is the best time for dental treatment because the woman will be able to sit most comfortably in the dental chair. Dental care, such as brushing with a fluoride toothpaste, is especially important during pregnancy. Periodontal disease has been linked to both preterm labor and low-birth-weight (LBW) infants. Emergency dental surgery is permissible; however, the mother must clearly understand the risks and benefits. Conscious relaxation is useful and may even help the woman get through any dental appointments, but it is not a reason to avoid them.

Intervention for the sexual abuse survivor is often not attempted by maternity and women's health nurses because of the concern about increasing the distress of the woman and the lack of expertise in counseling. What initial intervention is appropriate and most important in facilitating the woman's care? a. Initiating a referral to an expert counselor b. Setting limits on what the client discloses c. Listening and encouraging therapeutic communication skills d. Acknowledging the nurse's discomfort to the client as an expression of empathy

ANS: C The survivor needs support on many different levels, and a women's health nurse may be the first person to whom she relates her story. Therapeutic communication skills and listening are initial interventions. Referring this client to a counselor is an appropriate measure but not the most important initial intervention. A client should be allowed to disclose any information she feels the need to discuss. A nurse should provide a safe environment in which she can do so. Either verbal or nonverbal shock and horror reactions from the nurse are particularly devastating. Professional demeanor and professional empathy are essential.

Which consideration is essential for the nurse to understand regarding follow-up prenatal care visits? a. The interview portions become more intensive as the visits become more frequent over the course of the pregnancy. b. Monthly visits are scheduled for the first trimester, every 2 weeks for the second trimester, and weekly for the third trimester. c. During the abdominal examination, the nurse should be alert for supine hypotension. d. For pregnant women, a systolic BP of 130 mm Hg and a diastolic BP of 80 mm Hg is sufficient to be considered hypertensive.

ANS: C The woman lies on her back during the abdominal examination, possibly compressing the vena cava and aorta, which can cause a decrease in BP and a feeling of faintness. The interview portion of the follow-up examinations is less extensive than in the initial prenatal visits, during which so much new information must be gathered. Monthly visits are routinely scheduled for the first and second trimesters; visits increase to every 2 weeks at week 28 and to once a week at week 36. For pregnant women, hypertension is defined as a systolic BP of 140 mm Hg or higher and a diastolic BP of 90 mm Hg or higher.

A couple comes in for an infertility workup, having attempted to achieve pregnancy for 2 years. The woman, 37 years of age, has always had irregular menstrual cycles but is otherwise healthy. The man has fathered two children from a previous marriage and had a vasectomy reversal 2 years ago. The man has had two normal semen analyses, but the sperm seem to be clumped together. What additional testing is needed? a. Testicular biopsy b. Antisperm antibodies c. Serum prolactin level d. Examination for testicular infection

ANS: C This scenario does not indicate that the woman has had any testing related to her irregular menstrual cycles. Hormone analysis is performed to assess endocrine function of the hypothalamic-pituitary-ovarian axis when menstrual cycles are absent or irregular. Determining the blood levels of prolactin, FSH, luteinizing hormone (LH), estradiol, progesterone, and thyroid hormones may be necessary to diagnose the cause of the woman's irregular menstrual cycles. A testicular biopsy is indicated only in cases of azoospermia (no sperm cells) or severe oligospermia (low number of sperm cells). Although unlikely to be the case because the husband has already produced children, antisperm antibodies may be produced by the man against his own sperm. Examination for testicular infection would be performed before semen analysis. Furthermore, infection would affect spermatogenesis.

A 27-year-old pregnant woman had a preconceptual body mass index (BMI) of 19. What is this client's total recommended weight gain during pregnancy? a. 20 kg (44 lb) b. 16 kg (35 lb) c. 12.5 kg (27.5 lb) d. 10 kg (22 lb)

ANS: C This woman has a normal BMI and should gain 11.5 to 16 kg during her pregnancy. A weight gain of 20 kg (44 lb) is unhealthy for most women; a weight gain of 16 kg (35 lb) is at the high end of the range of weight this woman should gain in her pregnancy; and a weight gain of 10 kg (22 lb) is appropriate for an obese woman. This woman has a normal BMI, which indicates that her weight is average.

A woman will be taking oral contraceptives using a 28-day pack. What advice should the nurse provide to protect this client from an unintended pregnancy? a. Limit sexual contact for one cycle after starting the pill. b. Use condoms and foam instead of the pill for as long as the client takes an antibiotic. c. Take one pill at the same time every day. d. Throw away the pack and use a backup method if two pills are missed during week 1 of her cycle.

ANS: C To maintain adequate hormone levels for contraception and to enhance compliance, clients should take oral contraceptives at the same time each day. If contraceptives are to be started at any time other than during normal menses or within 3 weeks after birth or an abortion, then another method of contraception should be used through the first week to prevent the risk of pregnancy. Taken exactly as directed, oral contraceptives prevent ovulation, and pregnancy cannot occur. No strong pharmacokinetic evidence indicates a link between the use of broad-spectrum antibiotics and altered hormonal levels in oral contraceptive users. If the client misses two pills during week 1, then she should take two pills a day for 2 days and finish the package and use a backup contraceptive method for the next 7 consecutive days.

A pregnant woman at 10 weeks of gestation jogs three or four times per week. She is concerned about the effect of the exercise on the fetus. Which guidance should the nurse provide? a. "You don't need to modify your exercising any time during your pregnancy." b. "Stop exercising because it will harm the fetus." c. "You may find that you need to modify your exercise to walking later in your pregnancy, around the seventh month." d. "Jogging is too hard on your joints; switch to walking now."

ANS: C Typically, running should be replaced with walking around the seventh month of pregnancy. The nurse should inform the woman that she may need to reduce her exercise level as the pregnancy progresses. Physical activity promotes a feeling of well-being in pregnant women. It improves circulation, promotes relaxation and rest, and counteracts boredom. Simple measures should be initiated to prevent injuries, such as warm-up and stretching exercises to prepare the joints for more strenuous exercise.

A woman arrives at the clinic for a pregnancy test. Her last menstrual period (LMP) was February 14, 2019. What is the client's expected date of birth (EDB)? a. September 17, 2019 b. November 7, 2019 c. November 21, 2019 d. December 17, 2019

ANS: C Using the Nägele's rule, the EDB is calculated by subtracting 3 months from the month of the LMP and adding 7 days + 1 year to the day of the LMP. Therefore, with an LMP of February 14, 2019, her due date is November 21, 2019. September 17, 2019 is too short a period to complete a normal pregnancy. Using the Nägele's rule, an EDB of November 7, 2019, is 2 weeks early. December 17, 2019 is almost a month past the correct EDB

A couple is attempting to cope with an infertility problem. They want to know what they can do to preserve their emotional equilibrium. What is the nurse's most appropriate response? a. "Tell your friends and family so that they can help you." b ."Talk only to other friends who are infertile, because only they can help." c. "Get involved with a support group. I'll give you some names." d. "Start adoption proceedings immediately, because adopting an infant can be very difficult."

ANS: C Venting negative feelings may unburden the couple. A support group may provide a safe haven for the couple to share their experiences and gain insight from others' experiences. Although talking about their feelings may unburden them of negative feelings, infertility can be a major stressor that affects the couple's relationships with family and friends. Limiting their interactions to other infertile couples may be a beginning point for addressing psychosocial needs. However, depending on where the other couple is in their own recovery process, limiting their interactions may not be of assistance to them. Telling the couple to start adoption proceedings immediately is not supportive of the psychosocial needs of this couple and may be detrimental to their well-being.

During the initial visit with a client who is beginning prenatal care, which action should be the highest priority for the nurse? a. The first interview is a relaxed, get-acquainted affair during which the nurse gathers some general impressions of his or her new client. b. If the nurse observed handicapping conditions, he or she should be sensitive and not inquire about them because the client will do that in her own time. c. The nurse should be alert to the appearance of potential parenting problems, such as depression or lack of family support. d. Because of legal complications, the nurse should not ask about illegal drug use; that is left to the physician.

ANS: C Besides these potential problems, the nurse needs to be alert to the woman's attitude toward keeping regular health care appointments. If the client lacks insurance, then the nurse may be able to direct her to resources that provide assistance for pregnant women (i.e., Women, Infants, and Children [WIC]; Medicaid). The initial interview needs to be planned, purposeful, and focused on specific content. A lot of ground must be covered. The nurse must be sensitive to special problems; he or she should inquire because discovering individual needs is important. A client with a chronic or handicapping condition might forget to mention it because she has adapted to it. Obtaining information on drug use is important and can be confidentially done. Actual testing for drug use requires the client's consent.

Which procedure falls into the category of micromanipulation techniques of the follicle? (Select all that apply.) a. Intrauterine insemination b. Preimplantation genetic diagnosis c. Intracytoplasmic sperm injection (ISCI). d. Assisted hatching e. IVF-ET

ANS: C, D ISCI makes it possible to achieve fertilization even with a few or poor-quality sperm by introducing sperm beneath the zone pellucid into the egg. Another micromanipulation technique is assisted hatching. An infrared laser breaks through the thick or tough zone pellucid, enabling the blastocyst to hatch.

During pregnancy, many changes occur as a direct result of the presence of the fetus. Which of these adaptations meet this criterion? (Select all that apply.) a. Leukorrhea b. Development of the operculum c. Quickening d. Ballottement e. Lightening

ANS: C, D, E Quickening is the first recognition of fetal movements or "feeling life." Quickening is often described as a flutter and is felt earlier in the multiparous woman than in the primiparous woman. Passive movement of the unengaged fetus is referred to as ballottement. Lightening occurs when the fetus begins to descend into the pelvis and occurs 2 weeks before labor in the nulliparous woman and at the start of labor in the multiparous woman. Leukorrhea is a white or slightly gray vaginal discharge that develops in response to cervical stimulation by estrogen and progesterone. Mucus fills the cervical canal creating a plug otherwise known as the operculum. The operculum acts as a barrier against bacterial invasion during the pregnancy.

A pregnant couple has formulated a birth plan and is reviewing it with the nurse at an expectant parent's class. Which aspect of their birth plan should be considered potentially unrealistic and require further discussion with the nurse? a. "My partner and I have agreed that my sister will be my coach because he becomes anxious with regard to medical procedures and blood. He will be nearby and check on me every so often to make sure everything is okay." b. "We plan to use the techniques taught in the Lamaze classes to reduce the pain experienced during labor." c. "We want the labor and birth to take place in a birthing room. My partner will come in the minute the baby is born." d. "Regardless of the circumstances, we do not want the fetal monitor used during labor because it will interfere with movement and doing effleurage."

ANS: D Because monitoring is essential to assess fetal well-being, fetal monitoring is not a factor that can be determined by the couple. The nurse should fully explain its importance. The option for intermittent electronic monitoring could be explored if this is a low-risk pregnancy and as long as labor is normally progressing. The birth plan is a tool with which parents can explore their childbirth options; however, the plan must be viewed as tentative. Having the woman's sister as her coach with her husband nearby is an acceptable request for a laboring woman. Using breathing techniques to alleviate pain is a realistic part of a birth plan. Not all fathers are able to be present during the birth; however, this couple has made a realistic plan that works for their specific situation.

Pregnancy hormones prepare the vagina for stretching during labor and birth. Which change related to the pelvic viscera should the nurse share with the client? a. Because of several changes in the cervix, abnormal Papanicolaou (Pap) tests are easier to evaluate. b. Quickening is a technique of palpating the fetus to engage it in passive movement. c. The deepening color of the vaginal mucosa and cervix (Chadwick sign) usually appears in the second trimester or later as the vagina prepares to stretch during labor. d. Increased vascularity of the vagina increases sensitivity and may lead to a high degree of arousal, especially in the second trimester.

ANS: D Increased sensitivity and increased interest in sex sometimes go together and frequently occur during the second trimester. These cervical changes make evaluation of abnormal Pap tests more difficult. Quickening is the first recognition of fetal movements by the mother. Ballottement is a technique used to palpate the fetus. The Chadwick sign appears from the 6 to 8 weeks of gestation.

Nutrition is an alterable and important preventive measure for a variety of potential problems such as low birth weight and prematurity. While completing the physical assessment of the pregnant client, the nurse can evaluate the client's nutritional status by observing a number of physical signs. Which physical sign indicates to the nurse that the client has unmet nutritional needs? a. Normal heart rate, rhythm, and blood pressure b. Bright, clear, and shiny eyes c. Alert and responsive with good endurance d. Edema, tender calves, and tingling

ANS: D The physiologic changes of pregnancy may complicate the interpretation of physical findings. Lower extremity edema often occurs when caloric and protein deficiencies are present; however, edema in the lower extremities may also be a common physical finding during the third trimester. Completing a thorough health history and physical assessment and requesting further laboratory testing, if indicated, are essential for the nurse. The malnourished pregnant client may display rapid heart rate, abnormal rhythm, enlarged heart, and elevated blood pressure. A client receiving adequate nutrition will have bright, shiny eyes with no sores and moist, pink membranes. Pale or red membranes, dryness, infection, dull appearance of the cornea, or blue sclerae are signs of poor nutrition. A client who is alert and responsive with good endurance is well nourished. A listless, cachectic, easily fatigued, and tired presentation would be an indication of a poor nutritional status.

Which action is the priority for the nurse who is assessing the influence of culture on a client's diet? a. Evaluate the client's weight gain during pregnancy. b. Assess the socioeconomic status of the client. c. Discuss the four food groups with the client. d. Identify food preferences and methods of food preparation

ANS: D Understanding the client's food preferences and how she prepares food will assist the nurse in determining whether the client's culture is adversely affecting her nutritional intake. An evaluation of a client's weight gain during pregnancy should be included for all clients, not only for clients from different cultural backgrounds. The socioeconomic status of the client may alter the nutritional intake but not the cultural influence. Teaching the food groups to the client should come after assessing her food preferences.

A woman has come to the clinic for preconception counseling because she wants to start trying to get pregnant. Which guidance should she expect to receive? a. "Discontinue all contraception now." b."Lose weight so that you can gain more during pregnancy." c. "You may take any medications you have been regularly taking." d. "Make sure you include adequate folic acid in your diet."

ANS: D A healthy diet before conception is the best way to ensure that adequate nutrients are available for the developing fetus. A woman's folate or folic acid intake is of particular concern in the periconception period. Neural tube defects are more common in infants of women with a poor folic acid intake. Depending on the type of contraception that she has been using, discontinuing all contraception at this time may not be appropriate. Advising this client to lose weight now so that she can gain more during pregnancy is also not appropriate advice. Depending on the type of medications the woman is taking, continuing to take them regularly may not be appropriate.

Which pregnant woman should strictly follow weight gain recommendations during pregnancy? a. Pregnant with twins b. In early adolescence c. Shorter than 62 inches or 157 cm d. Was 20 pounds overweight before pregnancy

ANS: D A weight gain of 5 to 9 kg will provide enough nutrients for the fetus. Overweight and obese women should be advised to lose weight before conception to achieve the best pregnancy outcomes. A higher weight gain in twin gestations may help prevent low birth weights. Adolescents need to gain weight toward the higher acceptable range, which provides for their own growth, as well as for fetal growth. In the past, women of short stature were advised to restrict their weight gain; however, evidence to support these guidelines has not been found.

A 21-year-old client exhibits a greenish, copious, and malodorous discharge with vulvar irritation. A speculum examination and wet smear are performed to help confirm the diagnosis. Which condition is this client most likely experiencing? a. Bacterial vaginosis b. Candidiasis c. Yeast infection d. Trichomoniasis

ANS: D Although uncomfortable, a speculum examination is always performed, and a wet smear obtained if the client exhibits symptoms of trichomoniasis. The presence of many white blood cell protozoa is a positive finding for trichomoniasis. A normal saline test is used to test for bacterial vaginosis. A potassium hydroxide preparation is used to test for candidiasis. Yeast infection is the common name for candidiasis, for which the test is a potassium hydroxide preparation.

29. While assessing the vital signs of a pregnant woman in her third trimester, the client reports feeling faint, dizzy, and agitated. Which nursing intervention is appropriate? a. Have the patient stand up, and then retake her BP. b.Have the patient sit down, and then hold her arm in a dependent position. c. Have the patient lie supine for 5 minutes, and then recheck her BP on both arms. d. Have the patient turn to her left side, and then recheck her BP in 5 minutes.

ANS: D BP is affected by maternal position during pregnancy. The supine position may cause occlusion of the vena cava and descending aorta. Turning the pregnant woman to a lateral recumbent position alleviates pressure on the blood vessels and quickly corrects supine hypotension. Pressures are significantly higher when the client is standing. This option causes an increase in systolic and diastolic pressures. The arm should be supported at the same level of the heart. The supine position may cause occlusion of the vena cava and descending aorta, creating hypotension.

What is the importance of obtaining informed consent when educating a client regarding contraceptive methods? a. Contraception is an invasive procedure that requires hospitalization. b. The method may require a surgical procedure to insert a device. c. The contraception method chosen may be unreliable. d. The method chosen has potentially dangerous side effects.

ANS: D Being aware of the potential side effects is informed decision about the use of contraceptives. The only contraceptive method that is a surgical procedure and requires hospitalization is sterilization. Some methods have greater efficacy than others, and this efficacy should be included in the teaching. for couples who are making an

Which condition is likely to be a psychologic consequence of continued physical and psychological abuse? Select all that apply. a. Substance abuse b. Posttraumatic stress disorder (PTSD) c. Eating disorders d. CHOICE BLANK e. Bipolar disorder f. General anxiety

ANS: D Bipolar disorder is a specific illness (also known as manic depressive disorder) not related to abuse. Substance abuse is a common method of coping with long-term abuse. The abuser is also more likely to use alcohol and other chemical substances. PTSD is the most prevalent mental health sequela of long-term abuse. The traumatic event is persistently re-experienced through distress recollection and dreams. Eating disorders, depression, psychologic-physiologic illness, and anxiety reactions are all mental health problems associated with repeated abuse.

Which structure is responsible for oxygen and carbon dioxide transport to and from the maternal bloodstream? a. Decidua basalis b. Blastocyst c. Germ layer d. Chorionic villi

ANS: D Chorionic villi are fingerlike projections that develop out of the trophoblast and extend into the blood-filled spaces of the endometrium. The villi obtain oxygen and nutrients from the maternal bloodstream and dispose carbon dioxide and waste products into the maternal blood. The decidua basalis is the portion of the decidua (endometrium) under the blastocyst where the villi attach. The blastocyst is the embryonic development stage after the morula; implantation occurs at this stage. The germ layer is a layer of the blastocyst.

A man smokes two packs of cigarettes a day. He wants to know if smoking is contributing to the difficulty he and his wife are having getting pregnant. Which guidance should the nurse provide? a."Your sperm count seems to be okay in the first semen analysis." b. "Only marijuana cigarettes affect sperm count." c. "Although smoking has no effect on sperm count, it can give you lung cancer." d. "Smoking can reduce the quality of your sperm."

ANS: D Cigarette smoking has detrimental effects on sperm and has been associated with abnormal sperm, a decreased number of sperm, and chromosomal damage. The nurse may suggest a smoking cessation program to increase the fertility of the male partner. Sperm counts vary from day to day and are dependent on emotional and physical status and sexual activity. Therefore, a single analysis may be inconclusive. A minimum of two analyses must be performed several weeks apart to assess male fertility. Marijuana use may depress the number and motility of sperm. Smoking is indeed a causative agent for lung cancer.

Which statement regarding the term contraceptive failure rate is the most accurate? a. The contraceptive failure rate refers to the percentage of users expected to have an accidental pregnancy over a 5-year span. b. It refers to the minimum rate that must be achieved to receive a government license. c. The contraceptive failure rate increases over time as couples become more careless. d. It varies from couple to couple, depending on the method and the users.

ANS: D Contraceptive effectiveness varies from couple to couple, depending on how well a contraceptive method is used and how well it suits the couple. The contraceptive failure rate measures the likelihood of accidental pregnancy in the first year only. Failure rates decline over time because users gain experience.

A woman who is 8 months pregnant asks the nurse, "Does my baby have any antibodies to fight infection?" What is the most appropriate response by the nurse? a. "Your baby has all the immunoglobulins necessary: immunoglobulin G (IgG), immunoglobulin M (IgM), and immunoglobulin A (IgA)." b. "Your baby won't receive any antibodies until he is born and you breastfeed him." c. "Your baby does not have any antibodies to fight infection." d. "Your baby has IgG and IgM."

ANS: D During the third trimester, IgG is the only immunoglobulin that crosses the placenta; it provides passive acquired immunity to specific bacterial toxins. However, the fetus produces IgM by the end of the first trimester. IgA immunoglobulins are not produced by the baby. Therefore, by the third trimester, the fetus has both IgG and IgM. Breastfeeding supplies the newborn infant with IgA.

A woman in the 34th week of pregnancy reports that she is very uncomfortable because of heartburn. Which recommendation would be appropriate for this client? a. Substitute other calcium sources for milk in her diet. b. Lie down after each meal. c. Reduce the amount of fiber she consumes. d. Eat five small meals daily.

ANS: D Eating small, frequent meals may help with heartburn, nausea, and vomiting. Substituting other calcium sources for milk, lying down after eating, and reducing fiber intake are inappropriate dietary suggestions for all pregnant women and do not alleviate heartburn.

6. A woman is 3 months pregnant. At her prenatal visit she tells the nurse that she does not know what is happening; one minute she is happy that she is pregnant and the next minute she cries for no reason. Which response by the nurse is most appropriate? a. "Don't worry about it; you'll feel better in a month or so." b. "Have you talked to your husband about how you feel?" c. "Perhaps you really don't want to be pregnant." d. "Hormone changes during pregnancy commonly result in mood swings."

ANS: D Explaining that hormone changes can result in mood swings is an accurate statement and the most appropriate response by the nurse. Telling the woman not to worry dismisses her concerns and is not the most appropriate response. Although the woman should be encouraged to share her feelings, asking if she has spoken to her husband about them is not the most appropriate response and does not provide her with a rationale for the psychosocial dynamics of her pregnancy. Suggesting that the woman does not want to be pregnant is completely inappropriate and deleterious to the psychologic well-being of the woman. Hormonal and metabolic adaptations often cause mood swings in pregnancy. The woman's responses are normal. She should be reassured about her feelings.

hCG is an important biochemical marker for pregnancy and therefore the basis for many tests. Which statement regarding hCG is true? a. hCG can be detected as early as weeks after conception. b. hCG levels gradually and uniformly increase throughout pregnancy. c. Significantly lower-than-normal increases in the levels of hCG may indicate a postdate pregnancy. d. Higher-than-normal levels of hCG may indicate an ectopic pregnancy or Down syndrome.

ANS: D Higher hCG levels also could be a sign of a multiple gestation. hCG can be detected as early as 7 to 10 days after conception. The hCG levels fluctuate during pregnancy, peaking, declining, stabilizing, and then increasing again. Abnormally slow increases may indicate impending miscarriage.

Which vitamins or minerals may lead to congenital malformations of the fetus if taken in excess by the mother? a. Zinc b. Vitamin D c. Folic acid d. Vitamin A

ANS: D If taken in excess, vitamin A causes a number of problems. An analog of vitamin A appears in prescribed acne medications, which must not be taken during pregnancy. Zinc, vitamin D, and folic acid are all vital to good maternity and fetal health and are highly unlikely to be consumed in excess.

Which gastrointestinal alteration of pregnancy is a normal finding? a. Insufficient salivation (ptyalism) is caused by increases in estrogen. b. Acid indigestion (pyrosis) begins early but declines throughout pregnancy. c. Hyperthyroidism often develops (temporarily) because hormone production increases. d. Nausea and vomiting rarely have harmful effects on the fetus and may be beneficial.

ANS: D Normal nausea and vomiting rarely produce harmful effects and may be less likely to result in miscarriage or preterm labor. Ptyalism is excessive salivation that may be caused by a decrease in unconscious swallowing or by stimulation of the salivary glands. Pyrosis begins as early as the first trimester and intensifies through the third trimester. Increased hormone production does not lead to hyperthyroidism in pregnant women.

Nurses are often the first health care professional with whom a woman comes into contact after being sexually assaulted. Which statement best describes the initial care of a rape victim? a. All legal evidence is preserved during the physical examination. b. The victim appreciates the legal information; however, decides not to pursue legal proceedings. c. The victim states that she is going to advocate against sexual violence. d. The victim leaves the health care facility without feeling re-victimized.

ANS: D Nurses can assist clients through an examination that is as nontraumatic as possible with kindness, skill, and empathy. The initial care of the victim affects her recovery and decision to receive follow-up care. Preservation of all legal evidence is very important; however, this may not be the best measure in terms of evaluating the care of a rape victim. Offering legal information is not the best measure of evaluating the care that this victim received. The victim may well decide not to pursue legal proceedings. Advocating against sexual violence may be extremely therapeutic for the client after her initial recovery but not a measure of evaluating her care.

___ is part of the normal vaginal flora in 20% to 30% of healthy pregnant women. GBS has been associated with poor pregnancy outcomes and is an important factor in neonatal morbidity and mortality. Which finding is not a risk factor for neonatal Group B Streptococcus (GBS) infection? Positive prenatal Group B Streptococcus culture Preterm birth at 37 weeks or less of gestation Intrapartum maternal temperature of 38 C (100.4°F) or higher Premature rupture of membranes (PROM) lasting 12 hours

ANS: D PROM 18 hours or longer before the birth increases the risk for neonatal GBS infection. Positive prenatal culture is a risk factor for neonatal GBS infection. Preterm birth at 37 weeks or less of gestation remains a risk factor for neonatal GBS infection. Maternal temperature of 38 C or higher is also a risk factor for neonatal GBS infection.

Male fertility declines slowly after age 40 years; however, no cessation of sperm production analogous to menopause in women occurs in men. What condition is not associated with advanced paternal age? a. Autosomal dominant disorder b. Schizophrenia c. Autism spectrum disorder d. Down syndrome

ANS: D Paternal age older than 40 years is associated with an increased risk for autosomal dominant disorder, schizophrenia, and autism spectrum disorder in their offspring. Although Down syndrome can occur in any pregnancy, it is often associated with advanced maternal age.

Many pregnant women have questions regarding work and travel during pregnancy. Which education is a priority for the nurse to provide? a. Women should sit for as long as possible and cross their legs at the knees from time to time for exercise. b. Women should avoid seat belts and shoulder restraints in the car because they press on the fetus. c. Metal detectors at airport security checkpoints can harm the fetus if the woman passes through them several times. d. While working or traveling in a car or on an airplane, women should arrange to walk around at least every hour or so.

ANS: D Periodic walking helps prevent thrombophlebitis. Pregnant women should avoid sitting or standing for long periods and crossing the legs at the knees. Pregnant women must wear lap belts and shoulder restraints. The most common injury to the fetus comes from injury to the mother. Metal detectors at airport security checkpoints do not harm fetuses.

Which manifestation differentiates primary syphilis from secondary syphilis? a. Fever, headache, and malaise b. Widespread rash c. Identified by serologic testing d. Appearance of a chancre 2 months after infection

ANS: D Primary syphilis is characterized by a primary lesion (the chancre), which appears 5 to 90 days after infection. The chancre begins as a painless papule at the site of inoculation and erodes to form a nontender, shallow, and clean ulcer several millimeters to centimeters in size. Secondary syphilis occurs 6 weeks to 6 months after the appearance of the chancre and is characterized by a widespread maculopapular rash. The individual may also experience fever, headache, and malaise. Latent syphilis are those infections that lack clinical manifestations; however, they are detected by serologic testing.

Which hormone is responsible for the maturation of mammary gland tissue? a. Estrogen b. Testosterone c. Prolactin d. Progesterone

ANS: D Progesterone causes maturation of the mammary gland tissue, specifically acinar structures of the lobules. Estrogen increases the vascularity of the breast tissue. Testosterone has no bearing on breast development. Prolactin is produced after birth and released from the pituitary gland; it is produced in response to infant suckling and an emptying of the breasts.

Which hormone is essential for maintaining pregnancy? a. Estrogen b. hCG c. Oxytocin d. Progesterone

ANS: D Progesterone is essential for maintaining pregnancy; it does so by relaxing smooth muscles, which reduces uterine activity and prevents miscarriage. Estrogen plays a vital role in pregnancy, but it is not the primary hormone for maintaining pregnancy. hCG levels rise at implantation but decline after 60 to 70 days. Oxytocin stimulates uterine contractions.

A client states that she does not drink milk. Which foods should the nurse encourage this woman to consume in greater amounts to increase her calcium intake? a. Fresh apricots b. Canned clams c. Spaghetti with meat sauce d. Canned sardines

ANS: D Sardines are rich in calcium. Fresh apricots, canned clams, and spaghetti with meat sauce are not high in calcium.

Which sign or symptom is considered a first-trimester warning sign and should be immediately reported by the pregnant woman to her health care provider? a. Nausea with occasional vomiting b. Fatigue c. Urinary frequency d. Vaginal bleeding

ANS: D Signs and symptoms that must be reported include severe vomiting, fever and chills, burning on urination, diarrhea, abdominal cramping, and vaginal bleeding. These symptoms may be signs of complications of the pregnancy. Nausea with occasional vomiting is a normal first-trimester complaint. Although it may be worrisome or annoying to the mother, it is not usually an indication of a problem with the pregnancy. Fatigue is common during the first trimester. Because of physiologic changes that happen during pregnancy, clients should be taught that urinary frequency is normal.

Which statement regarding gamete intrafallopian transfer (GIFT) is most accurate? a. Semen is collected after laparoscopy. b. Women must have two normal fallopian tubes. c. Ovulation spontaneously occurs. d. Ova and sperm are transferred to one tube.

ANS: D Similar to in vitro fertilization (IVF), GIFT requires the woman to have at least one normal tube. Ovulation is induced, and the oocytes are aspirated during laparoscopy. Semen is collected before laparoscopy. The ova and sperm are then transferred to one uterine tube, permitting natural fertilization and cleavage.

A pregnant woman's diet may not meet her increased need for folates. Which food is a rich source of this nutrient? a. Chicken b. Cheese c. Potatoes d. Green leafy vegetables

ANS: D Sources of folates include green leafy vegetables, whole grains, fruits, liver, dried peas, and beans. Chicken and cheese are excellent sources of protein but are poor sources for folates. Potatoes contain carbohydrates and vitamins and minerals but are poor sources for folates.

Numerous changes in the integumentary system occur during pregnancy. Which change persists after birth? a. Epulis b. Chloasma c. Telangiectasia d. Striae gravidarum

ANS: D Striae gravidarum, or stretch marks, reflect a separation within the underlying connective tissue of the skin. They usually fade after birth, although they never completely disappear. An epulis is a red, raised nodule on the gums that easily bleeds; it disappears or shrinks after giving birth. Chloasma, or the mask of pregnancy, is a blotchy, brown hyperpigmentation of the skin over the cheeks, nose, and forehead, especially in dark-complexioned pregnant women. Chloasma usually fades after the birth. Telangiectasia, or vascular spiders, are tiny, star-shaped or branchlike, slightly raised, pulsating end-arterioles usually found on the neck, thorax, face, and arms. They occur because of elevated levels of circulating estrogen and usually disappear after birth.

A woman is using the basal body temperature (BBT) method of contraception. She calls the clinic and tells the nurse, "My period is due in a few days, and my temperature has not gone up." What is the nurse's most appropriate response? a. "This probably means that you're pregnant." b. "Don't worry; it's probably nothing." c. "Have you been sick this month?" d. "You probably didn't ovulate during this cycle."

ANS: D The absence of a temperature decrease most likely is the result of a lack of ovulation. Pregnancy cannot occur without ovulation, which is being measured using the BBT method. A comment such as, "Don't worry; it's probably nothing," discredits the client's concerns. Illness is most likely the cause of an increase in BBT.

A client currently uses a diaphragm and spermicide for contraception. She asks the nurse to explain the major differences between the cervical cap and the diaphragm. What is the most appropriate response by the nurse? a. "No spermicide is used with the cervical cap, so it's less messy." b. "The diaphragm can be left in place longer after intercourse." c. "Repeated intercourse with the diaphragm is more convenient." d. "The cervical cap can be safely used for repeated acts of intercourse without adding more spermicide later."

ANS: D The cervical cap can be inserted hours before sexual intercourse without the need for additional spermicide later. Spermicide should be used inside the cap as an additional chemical barrier. The cervical cap should remain in place for 6 hours after the last act of intercourse. Repeated intercourse with the cervical cap is more convenient because no additional spermicide is needed.

A blind woman has arrived for an examination. She appears nervous and says, "I've never had a pelvic examination." What response from the nurse would be most appropriate? a. "Being visually impaired must be very anxiety producing." b. "Try to relax. I'll be very gentle, and I promise not to hurt you." c. "Your anxiety is common. I was anxious when I first had a pelvic examination." d. "I'll let you touch each instrument I'll be using as I tell you how it will be used."

ANS: D The client who is visually impaired needs to be oriented to the examination room and needs a full explanation of what the examination entails before the nurse proceeds. The statement regarding her visual disability and anxiety does not address her concerns. The nurse should openly and directly communicate with sensitivity. Women who have physical disabilities should be respected and involved in the assessment and physical examination to the full extent of their abilities. Telling the client that she will not be hurt does not reflect respect or sensitivity. Although anxiety may be common, the nurse should not discuss her own issues nor compare them to the client's concerns.

Which part of the menstrual cycle includes the stimulated release of gonadotropin-releasing hormone (GnRH) and follicle-stimulating hormone (FSH)? a. Menstrual phase b. Endometrial cycle c. Ovarian cycle d. Hypothalamic-pituitary cycle

ANS: D The cyclic release of hormones is the function of the hypothalamus and pituitary glands. The menstrual cycle is a complex interplay of events that simultaneously occur in the endometrium, hypothalamus, pituitary glands, and ovaries. The endometrial cycle consists of four phases: menstrual phase, proliferative phase, secretory phase, and ischemic phase. The ovarian cycle remains under the influence of FSH and estrogen.

Nurses should be cognizant of what information regarding the non-contraceptive medical effects of combination oral contraceptives (COCs)? a. COCs can cause TSS if the prescription is wrong. b. Hormonal withdrawal bleeding is usually a little more profuse than in normal menstruation and lasts a week for those who use COCs. c. COCs increase the risk of endometrial and ovarian cancers. d. Effectiveness of COCs can be altered by some over-the-counter medications and herbal supplements.

ANS: D The effectiveness of COCs can be altered by some over-the-counter medications and herbal supplements. TSS can occur in some who use the diaphragm, but it is not a consequence of taking oral contraceptive pills. Hormonal withdrawal bleeding usually is lighter than in normal menstruation and lasts a couple of days. Oral contraceptive pills offer protection against the risk of endometrial and ovarian cancers.

A woman who is 14 weeks pregnant tells the nurse that she always had a glass of wine with dinner before she became pregnant. She has abstained during her first trimester and would like to know if it is safe for her to have a drink with dinner now. Which guidance should the nurse provide? a. "Since you're in your second trimester, there's no problem with having one drink with dinner." b. "One drink every night is too much. One drink three times a week should be fine." c. "Since you're in your second trimester, you can drink as much as you like." d. "Because no one knows how much or how little alcohol it takes to cause fetal problems, the best course is to abstain throughout your pregnancy."

ANS: D The statement "Because no one knows how much or how little alcohol it takes to cause fetal problems, the best course is to abstain throughout your pregnancy" is accurate. A safe level of alcohol consumption during pregnancy has not yet been established. Although the consumption of occasional alcoholic beverages may not be harmful to the mother or her developing fetus, complete abstinence is strongly advised.

What is the goal of a long-term treatment plan for an adolescent with an eating disorder? a. Managing the effects of malnutrition b. Establishing sufficient caloric intake c. Improving family dynamics d. Restructuring client perception of body image

ANS: D The treatment of eating disorders is initially focused on reestablishing physiologic homeostasis. Once body systems are stabilized, the next goal of treatment for eating disorders is maintaining adequate caloric intake. Although family therapy is indicated when dysfunctional family relationships exist, the primary focus of therapy for eating disorders is to help the adolescent cope with complex issues. The focus of treatment in individual therapy for an eating disorder involves restructuring cognitive perceptions about the

Which sexually transmitted infection (STI) can be successfully treated? a. Herpes simplex virus b. Acquired immunodeficiency syndrome (AIDS) c. Venereal warts d. Chlamydia

ANS: D The usual treatment for Chlamydia bacterial infection is doxycycline or azithromycin. Concurrent treatment of all sexual partners is needed to prevent recurrence. No known cure is available for HSV; therefore, the treatment focuses on pain relief and preventing secondary infections. Because no cure is known for AIDS, prevention and early detection are the primary focus of care management. HPV causes condylomata acuminata (venereal warts); no available treatment eradicates the virus.

What represents a typical progression through the phases of a woman's establishing a relationship with the fetus? a. Accepts the fetus as distinct from herself—accepts the biologic fact of pregnancy—has feelings of caring and responsibility. b. Fantasizes about the child's gender and personality—views the child as part of herself—becomes introspective. c. Views the child as part of herself—has feelings of well-being—accepts the biologic fact of the pregnancy. d. "I am pregnant"—"I am going to have a baby"—"I am going to be a mother."

ANS: D The woman first centers on herself as pregnant, then on the baby as an entity separate from herself, and then on her responsibilities as a mother. The expressions "I am pregnant," "I am going to have a baby," and "I am going to be a mother" sum up the progression through the three phases. In phase one, the woman views the child as part of herself and not as a separate being. This is only the first step of the progression through phases of attachment. Accepting the fetus as distinct from herself occurs during the second phase of emotional attachment. Fantasizing about the child's sex and personality based on fetal activity occurs during the third phase of attachment.

A woman who is 16 weeks pregnant has come in for a follow-up visit with her partner. To reassure the client regarding fetal well-being, which is the highest priority action for the nurse to perform? a. Assess the fetal heart tones with a Doppler stethoscope. b. Measure and document the girth of the woman's abdomen. c. Complete an ultrasound examination (sonogram) in a timely manner. d. Offer the woman and her partner the opportunity to listen to the fetal heart tones.

ANS: D To provide the parents with the greatest sense of reassurance, the nurse should offer to have the client and her significant other the chance to listen to their baby's heartbeat. A fetoscope can detect the fetal heart rate around 20 weeks of gestation. Doppler can detect the fetal heart rate between 10 and 12 weeks and should be performed as part of routine fetal assessment. Abdominal girth is not a valid measure for determining fetal well-being. Fundal height is an important measure that should be determined with precision, with the same technique and positioning of the client consistently used at every prenatal visit. Routine ultrasound examinations are recommended in early pregnancy; they date the pregnancy and provide useful information about the health of the fetus. However, they are not necessary at each prenatal visit.

To reassure and educate their pregnant clients regarding changes in their blood pressure, nurses should be cognizant of what? a. A blood pressure cuff that is too small produces a reading that is too low; a cuff that is too large produces a reading that is too high. b. Shifting the client's position and changing from arm to arm for different measurements produces the most accurate composite blood pressure reading at each visit. c. Systolic blood pressure slightly increases as the pregnancy advances; diastolic pressure remains constant. d. Compression of the iliac veins and inferior vena cava by the uterus contributes to hemorrhoids in the later stage of a term pregnancy.

ANS: D Compression of the iliac veins and inferior vena cava by the uterus contributes to hemorrhoids in the later stage of a term pregnancy. This compression also leads to varicose veins in the legs and vulva. The tightness of a blood pressure cuff that is too small produces a reading that is too high; similarly, the looseness of a cuff that is too large results in a reading that is too low. Because maternal positioning affects readings, blood pressure measurements should be obtained in the same arm and with the woman in the same position. The systolic blood pressure generally remains constant but may decline slightly as the pregnancy advances. The diastolic blood pressure first decreases and then gradually increases.

The BMI for a woman who is 51 kg before pregnancy and 1.57 m tall is _________.

ANS:20.7 A commonly used method of evaluating the appropriateness of weight for height is the BMI, which is calculated by the following formula.BMI = Weight in kg divided by the height in meters squaredBMI = 51 kg (1.57 m)2 51 2.47 = 20.69

The practice of the calendar rhythm method is based on the number of days in each menstrual cycle. The fertile period is determined after monitoring each cycle for 6 months. The beginning of the fertile period is estimated by subtracting 18 days from the longest cycle and 11 days from the shortest. If the woman's cycles vary in length from 24 to 30 days, then her fertile period would be day _____ through day ______.

ANS:6; 19 To avoid pregnancy, the couple must abstain from intercourse on days 6 through 19. Ovulation occurs on day 12 (plus or minus 2 days either way).

A woman arrives at the clinic for a pregnancy test. The first day of her LMP was September 10, 2014. Her expected date of birth (EDB) is __________.

ANS:June 17, 2015 Using the Nägele's rule, June 17, 2015, is the correct EDB. The EDB is calculated by subtracting 3 months from the first day of the LMP and adding 7 days + 1 year to the day of the LMP. Therefore, with an LMP of September 10, 2014: September 10, 2014 - 3 months = June 10, 2014 + 7 days = June 17, 2014 + 1 year = June 17, 2015.


Kaugnay na mga set ng pag-aaral

Chapt. 11 - Stratification by Gender

View Set

اساسيات التخطيط - الجزء الاول: التخط

View Set

History of ASL and Deaf Education in America

View Set